[obm-l] RE: [obm-l] Dúvidas!!

2015-11-26 Por tôpico Gustavo Henrique dos Santos
01. 26 litros de uma solução de álcool + solvente a 30% (ou 30 graus G.L.) 
contêm 26 * 0,30 = 7,8 litros de álcool.Logo, são 26,0 - 7,8 = 18,2 litros de 
solvente.É necessário acrescentar x litros de soluto para que (x + 26) - 0,35 * 
(x + 26) = 18,2, sendo x + 26 o volume finalPortanto, x +26 - 0,35 * x - 9,1 = 
18,2 ==> 0,65 * x + 16,9 = 18,2 ==> 0,65 * x = 1,3 ==> x = 2 litros.Logo, 2 
litros de álcool devem ser adicionados.
02. 90% das crianças estão gripadas. Logo, é provável que 90% * 8% = 7,2% das 
crianças estejam gripadas e tenham manchas vermelhas na pele, e 90% * 92% = 
82,8% das crianças estejam gripadas e NÃO tenham manchas vermelhas na pele.10% 
das crianças estão com rubéola. Logo, é provável que 10% * 95% = 9,5% das 
crianças estejam com rubéola e tenham manchas vermelhas na pele e 10% * 5% = 
0,5% das crianças estejam com rubéola e NÃO tenham manchas vermelhas na pele.Se 
a criança examinada pelo médico tem manchas vermelhas na pele, ela está dentre 
os 7,2% + 9,5% = 16,7% de prováveis crianças com manchas vermelhas na pele. 
Fazendo uma regra de três:
Rubéola: 9,5 - xQualquer doença: 16,7 
--- 100%
percebemos que x = 9,5 * 100% / 16,7. Logo, x é aproximadamente igual a 56,9%, 
que está mais próximo de 57%.Logo, há 57% de chances de que a criança tenha 
rubéola.
From: claudiot...@hotmail.com
To: obm-l@mat.puc-rio.br
Subject: [obm-l] Dúvidas!!
Date: Thu, 26 Nov 2015 02:21:43 +




01.Quantos litros de álcool devem ser adicionados a 26 litros de uma solução 
com 30% de álcool, para obtermos uma segunda solução com concentração de 35% de 
álcool?
02.Um médico foi chamado para examinar uma criança doente. Na vizinhança onde a 
criança mora, 90% das crianças estão gripadas, e os outros 10% estão com 
rubéola. Um sintoma comum de rubéola é o aparecimento de manchas vermelhas na 
pele, o que ocorre com probabilidade de 95%. No caso de gripe, manchas 
vermelhas na pele aparecem com probabilidade de 8%. Se, depois de examinar a 
criança, o médico observa que ela tem manchas vermelhas na pele, qual a 
probabilidade de a criança ter rubéola? Indique o valor inteiro mais próximo do 
valor obtido.
Agradeço Antecipadamente  
--

Esta mensagem foi verificada pelo sistema de antivírus e 

 acredita-se estar livre de perigo.




Este e-mail foi enviado por um computador sem vírus e protegido 
pelo Avast. www.avast.com   


  
-- 
Esta mensagem foi verificada pelo sistema de antivírus e
 acredita-se estar livre de perigo.



[obm-l] RE: [obm-l] RE: [obm-l] RE: [obm-l] Dúvidas

2015-03-06 Por tôpico Eduardo Henrique
Leia melhor a pergunta, e verifique que minha resposta é condizente. Note que 
em nenhum momento eu estou limitado a um grupo de 40 pessoas e no enunciado não 
fala que eu devo 'substituir' os homens por mulheres.

Att.

Eduardo

From: claudiot...@hotmail.com
To: obm-l@mat.puc-rio.br
Subject: [obm-l] RE: [obm-l] RE: [obm-l] Dúvidas
Date: Fri, 6 Mar 2015 13:13:06 +




Mas, como 100 mulheres devem ser retiradas , se no grupo tem no máximo 40 
pessoas??
Eu fiz deu 20 mulheres , não sei se estou errado.
Abraços

From: dr.dhe...@outlook.com
To: obm-l@mat.puc-rio.br
Subject: [obm-l] RE: [obm-l] Dúvidas
Date: Fri, 27 Feb 2015 05:28:16 +0300




Num grupo de 40 adultos, exatamente 30% são de sexo feminino. Há várias 
maneiras de se aumentar essa porcentagem, seja introduzindo mulheres no grupo 
,seja excluindo homens dele.

 a)Quantas mulheres devem ser introduzidas no grupo ,de modo que a porcentagem 
de mulheres passe para 80%? Há 40 pessoas no grupo, 30% mulheres, logo há 12 
mulheres e portanto 28 homens. Queremos 'colocar' mulheres para que o 
percentual chegue em 80%, dai então os 28 homens serão apenas 20% de um novo 
grupo maior. este grupo maior terá 140 pessoas (por que?). Logo você tem 140-28 
= 112 mulheres nesse novo grupo, mas 12 já estavam lá, logo introduziu 100 
mulheres.
 

b)Quantos homens devem ser excluídos do grupo, de modo que a porcentagem de 
mulheres passe para 80%?Você quer que 12 mulheres sejam 80% do novo grupo;
12 - 80%
x - 100% = x=15. Ora, você tinha 28 homens no grupo, sobraram 15, logo você 
retirou 13.

  
--

Esta mensagem foi verificada pelo sistema de antivírus e 

 acredita-se estar livre de perigo.   
--

Esta mensagem foi verificada pelo sistema de antivírus e 

 acredita-se estar livre de perigo.   
--

Esta mensagem foi verificada pelo sistema de antivírus e 

 acredita-se estar livre de perigo.   
-- 
Esta mensagem foi verificada pelo sistema de antivírus e
 acredita-se estar livre de perigo.



[obm-l] RE: [obm-l] RE: [obm-l] Dúvidas

2015-03-06 Por tôpico Cláudio Thor
Mas, como 100 mulheres devem ser retiradas , se no grupo tem no máximo 40 
pessoas??
Eu fiz deu 20 mulheres , não sei se estou errado.
Abraços

From: dr.dhe...@outlook.com
To: obm-l@mat.puc-rio.br
Subject: [obm-l] RE: [obm-l] Dúvidas
Date: Fri, 27 Feb 2015 05:28:16 +0300




Num grupo de 40 adultos, exatamente 30% são de sexo feminino. Há várias 
maneiras de se aumentar essa porcentagem, seja introduzindo mulheres no grupo 
,seja excluindo homens dele.

 a)Quantas mulheres devem ser introduzidas no grupo ,de modo que a porcentagem 
de mulheres passe para 80%? Há 40 pessoas no grupo, 30% mulheres, logo há 12 
mulheres e portanto 28 homens. Queremos 'colocar' mulheres para que o 
percentual chegue em 80%, dai então os 28 homens serão apenas 20% de um novo 
grupo maior. este grupo maior terá 140 pessoas (por que?). Logo você tem 140-28 
= 112 mulheres nesse novo grupo, mas 12 já estavam lá, logo introduziu 100 
mulheres.
 

b)Quantos homens devem ser excluídos do grupo, de modo que a porcentagem de 
mulheres passe para 80%?Você quer que 12 mulheres sejam 80% do novo grupo;
12 - 80%
x - 100% = x=15. Ora, você tinha 28 homens no grupo, sobraram 15, logo você 
retirou 13.

  
--

Esta mensagem foi verificada pelo sistema de antivírus e 

 acredita-se estar livre de perigo.   
--

Esta mensagem foi verificada pelo sistema de antivírus e 

 acredita-se estar livre de perigo.   
-- 
Esta mensagem foi verificada pelo sistema de antivírus e
 acredita-se estar livre de perigo.



[obm-l] RE: [obm-l] RE: [obm-l] RE: [obm-l] RE: [obm-l] Dúvidas

2015-03-06 Por tôpico Cláudio Thor
Ok!!
Entendi.
Obrigado.

From: dr.dhe...@outlook.com
To: obm-l@mat.puc-rio.br
Subject: [obm-l] RE: [obm-l] RE: [obm-l] RE: [obm-l] Dúvidas
Date: Fri, 6 Mar 2015 16:50:12 +0300




Leia melhor a pergunta, e verifique que minha resposta é condizente. Note que 
em nenhum momento eu estou limitado a um grupo de 40 pessoas e no enunciado não 
fala que eu devo 'substituir' os homens por mulheres.

Att.

Eduardo

From: claudiot...@hotmail.com
To: obm-l@mat.puc-rio.br
Subject: [obm-l] RE: [obm-l] RE: [obm-l] Dúvidas
Date: Fri, 6 Mar 2015 13:13:06 +




Mas, como 100 mulheres devem ser retiradas , se no grupo tem no máximo 40 
pessoas??
Eu fiz deu 20 mulheres , não sei se estou errado.
Abraços

From: dr.dhe...@outlook.com
To: obm-l@mat.puc-rio.br
Subject: [obm-l] RE: [obm-l] Dúvidas
Date: Fri, 27 Feb 2015 05:28:16 +0300




Num grupo de 40 adultos, exatamente 30% são de sexo feminino. Há várias 
maneiras de se aumentar essa porcentagem, seja introduzindo mulheres no grupo 
,seja excluindo homens dele.

 a)Quantas mulheres devem ser introduzidas no grupo ,de modo que a porcentagem 
de mulheres passe para 80%? Há 40 pessoas no grupo, 30% mulheres, logo há 12 
mulheres e portanto 28 homens. Queremos 'colocar' mulheres para que o 
percentual chegue em 80%, dai então os 28 homens serão apenas 20% de um novo 
grupo maior. este grupo maior terá 140 pessoas (por que?). Logo você tem 140-28 
= 112 mulheres nesse novo grupo, mas 12 já estavam lá, logo introduziu 100 
mulheres.
 

b)Quantos homens devem ser excluídos do grupo, de modo que a porcentagem de 
mulheres passe para 80%?Você quer que 12 mulheres sejam 80% do novo grupo;
12 - 80%
x - 100% = x=15. Ora, você tinha 28 homens no grupo, sobraram 15, logo você 
retirou 13.

  
--

Esta mensagem foi verificada pelo sistema de antivírus e 

 acredita-se estar livre de perigo.   
--

Esta mensagem foi verificada pelo sistema de antivírus e 

 acredita-se estar livre de perigo.   
--

Esta mensagem foi verificada pelo sistema de antivírus e 

 acredita-se estar livre de perigo.   
--

Esta mensagem foi verificada pelo sistema de antivírus e 

 acredita-se estar livre de perigo.   
-- 
Esta mensagem foi verificada pelo sistema de antivírus e
 acredita-se estar livre de perigo.



Re: [obm-l] RE: [obm-l] RE: [obm-l] Dúvidas

2015-03-06 Por tôpico nepier
 

Em 06/03/2015 10:13, Cláudio Thor escreveu: 

 Mas, como 100 mulheres devem ser retiradas , se no grupo tem no máximo 40 
 pessoas?? 
 
 Eu fiz deu 20 mulheres , não sei se estou errado. 
 
 Abraços
 
 -
 From: dr.dhe...@outlook.com
 To: obm-l@mat.puc-rio.br
 Subject: [obm-l] RE: [obm-l] Dúvidas
 Date: Fri, 27 Feb 2015 05:28:16 +0300
 
 NUM GRUPO DE 40 ADULTOS, EXATAMENTE 30% SÃO DE SEXO FEMININO. HÁ VÁRIAS 
 MANEIRAS DE SE AUMENTAR ESSA PORCENTAGEM, SEJA INTRODUZINDO MULHERES NO GRUPO 
 ,SEJA EXCLUINDO HOMENS DELE. 
 
   
 
   
 
   
 
 A)QUANTAS MULHERES DEVEM SER INTRODUZIDAS NO GRUPO ,DE MODO QUE A PORCENTAGEM 
 DE MULHERES PASSE PARA 80%? 
 
 Há 40 pessoas no grupo, 30% mulheres, logo há 12 mulheres e portanto 28 
 homens. Queremos 'colocar' mulheres para que o percentual chegue em 80%, dai 
 então os 28 homens serão apenas 20% de um novo grupo maior. este grupo maior 
 terá 140 pessoas (por que?). Logo você tem 140-28 = 112 mulheres nesse novo 
 grupo, mas 12 já estavam lá, logo introduziu 100 mulheres.
 
   
 
   
 
   
 
 B)QUANTOS HOMENS DEVEM SER EXCLUÍDOS DO GRUPO, DE MODO QUE A PORCENTAGEM DE 
 MULHERES PASSE PARA 80%? Você quer que 12 mulheres sejam 80% do novo grupo;
 12 - 80%
 x - 100% = x=15. Ora, você tinha 28 homens no grupo, sobraram 15, logo você 
 retirou 13.
 
 -- 
 Esta mensagem foi verificada pelo sistema de antivírus e 
 acredita-se estar livre de perigo. 
 -- 
 Esta mensagem foi verificada pelo sistema de antivírus e 
 acredita-se estar livre de perigo. 
 -- 
 Esta mensagem foi verificada pelo sistema de antivírus e 
 acredita-se estar livre de perigo.

 B)QUANTOS HOMENS DEVEM SER EXCLUÍDOS DO GRUPO, DE MODO QUE A PORCENTAGEM DE 
 MULHERES PASSE PARA 80%? Você quer que 12 mulheres sejam 80% do novo grupo;
 12 - 80%
 x - 100% = x=15. Ora, 15 é o novo grupo (H + M) como 12 são mulheres, então 
 3 são Homens. Se você tinha 28 homens no grupo anterior e no novo tem 3, você 
 retirou 25 Homens.
 
-- 
Esta mensagem foi verificada pelo sistema de antiv�rus e
 acredita-se estar livre de perigo.



[obm-l] RE: [obm-l] Dúvidas

2015-03-01 Por tôpico JOE JUNIOR
primeiro veja que temos 28 homens e 12 mulheres.
resp a)
para que a porcentagem de mulheres passe para 80% a de homens tem que ser 
20%.28 está para 20%, assim como x está pra 80%. Sendo x o nº total de mulheres 
no grupo.fazendo a regrinha de três encontramos x = 112 mulheres no novo grupo. 
logo 112 - 12 = 100 mulheres adicionadas
resp b)seguindo um raciocínio parecido teremos que12 está para 80% assim como y 
está para 20. Sendo y o nº total de homens no grupo.fazendo a regrinha de três 
encontramos y = 3 homens no grupo. Logo 28 - 3 = 25 homens excluídos do grupo.


Num grupo de 40 adultos, exatamente 30% são de sexo feminino. Há várias 
maneiras de se aumentar essa porcentagem, seja introduzindo mulheres no grupo 
,seja excluindo homens dele.
a)Quantas mulheres devem ser introduzidas no grupo ,de modo que a porcentagem 
de mulheres passe para 80%? 
b)Quantos homens devem ser excluídos do grupo, de modo que a porcentagem de 
mulheres passe para 80%?

  
--

Esta mensagem foi verificada pelo sistema de antivírus e 

 acredita-se estar livre de perigo.   
-- 
Esta mensagem foi verificada pelo sistema de antivírus e
 acredita-se estar livre de perigo.



[obm-l] RE: [obm-l] Dúvidas

2015-02-26 Por tôpico Eduardo Henrique
Ops, na última conta ali eu cometi um erro, me perdoe.

x=15 implica que o novo grupo tem 15 pessoas e como 12 são mulheres, sobraram 
apenas 3 homens, logo 25 homens foram retirados.

From: dr.dhe...@outlook.com
To: obm-l@mat.puc-rio.br
Subject: RE: [obm-l] Dúvidas
Date: Fri, 27 Feb 2015 05:28:16 +0300




Num grupo de 40 adultos, exatamente 30% são de sexo feminino. Há várias 
maneiras de se aumentar essa porcentagem, seja introduzindo mulheres no grupo 
,seja excluindo homens dele.

 a)Quantas mulheres devem ser introduzidas no grupo ,de modo que a porcentagem 
de mulheres passe para 80%? Há 40 pessoas no grupo, 30% mulheres, logo há 12 
mulheres e portanto 28 homens. Queremos 'colocar' mulheres para que o 
percentual chegue em 80%, dai então os 28 homens serão apenas 20% de um novo 
grupo maior. este grupo maior terá 140 pessoas (por que?). Logo você tem 140-28 
= 112 mulheres nesse novo grupo, mas 12 já estavam lá, logo introduziu 100 
mulheres.
 

b)Quantos homens devem ser excluídos do grupo, de modo que a porcentagem de 
mulheres passe para 80%?Você quer que 12 mulheres sejam 80% do novo grupo;
12 - 80%
x - 100% = x=15. Ora, você tinha 28 homens no grupo, sobraram 15, logo você 
retirou 13.

  
--

Esta mensagem foi verificada pelo sistema de antivírus e 

 acredita-se estar livre de perigo. 
  
-- 
Esta mensagem foi verificada pelo sistema de antivírus e
 acredita-se estar livre de perigo.



[obm-l] RE: [obm-l] Dúvidas em combinatória

2012-06-11 Por tôpico marcone augusto araújo borges

Obrigado,Douglas.
Uma problema bem parecido: Uma escada tem n degraus.Voce sobe tomando um ou 
dois a cada vez.De quantas maneiras voce pode subir?

 



Date: Mon, 11 Jun 2012 15:42:45 -0300
From: douglas.olive...@grupoolimpo.com.br
To: obm-l@mat.puc-rio.br
Subject: Re: [obm-l] Dúvidas em combinatória


Problema interessantíssimo, não tinha parado pra fazer até que percebi algo..
se voce for analisando a medida que os elementos crescem no conjunto perceba:
{}  1
{1}--- 2
{1,2}---3
{1,2,3}---5
{1,2,3,4}---8
...
os números que aparecem são os de fibonacci e analisando a sua resolução, voce 
mesmo chegaria no teorema de lucas f_n+1=Cn,0 + Cn-1,1 +Cn-2,2 +...Cn-j,j onde 
j é o maior inteiro menor ou igual a n/2, o que responde sua pergunta sobre n/2.
logo é só montar a recorrência e escrever a fórmula de binet.
Espero ter ajudado.
Douglas Oliveira!!!
 
 
On Mon, 4 Jun 2012 13:38:50 +, marcone augusto araújo borges wrote:

1)Quantos subconjuntos do conjunto {1,2,...,n} não contêm dois inteiros 
consecutivos?
 
O vazio seria um deles
Com 1 elemento:n subconjuntos
Com 2 elementos:Cn-1,2
Com 3 elementos:Cn-2,3
  .
  .
  .
Com n/2 elementos(se n é par):???
Eu pensei C(n/2 + 1,n/2) = n/2 + 1...mas isso é muito estranho,pois,se n = 
10,por exemplo,só há 2 subconjuntos de 5 elementos que não contêm dois inteiros 
consecutivos...
è necessario mesmo separar em 2 casos,n par e n ímpar?
 
2)Qual o argumento combinatório para mostrar que Cn,2 + Cn+1,2 = n^2?
 
Desde já agradeço.
 
 
  

[obm-l] RE: [obm-l] Dúvidas- Logica Matemática

2012-05-08 Por tôpico marcone augusto araújo borges

A questão 2,eu acho,seria passível de anulação,sim.

 



From: vanessani...@hotmail.com
To: obm-l@mat.puc-rio.br
Subject: [obm-l] Dúvidas- Logica Matemática
Date: Tue, 8 May 2012 03:07:40 +




 1-  Um professor de Lógica, recém chegado a este país, é informado por um 
nativo que glup e plug, na língua local, significam sim e não mas o professor 
não sabe se o nativo que o informou é verd ou falc. Então ele se aproxima de 
três outros nativos que estavam conversando juntos e faz a cada um deles duas 
perguntas:1ª Os outros dois são verds?2ª Os outros dois são falcs?A primeira 
pergunta é respondida com glup pelos três mas à segunda pergunta os dois 
primeiros responderam glup e o terceiro respondeu plug.Assim, o professor pode 
concluir que:a) todos são verds;b) todos são falcs;c) somente um dos três 
últimos é falc e glup significa não;d) somente um dos três últimos é verd e 
glup significa sim;e) há dois verds e glup significa sim.
Dúvida no gabarito
2-Uma cafeteira automática aceita apenas moedas de 5, 10 ou 25 centavos e não 
devolve troco. Se, feito nessa máquina, cada cafezinho custa 50 centavos, de 
quantos modos podem ser usadas essas moedas para pagá-lo? 
(A) 13 
(B) 12 
(C) 11 
(D) 10 
(E) 9
O gabarito marca como 10 maneiras, até ai ok, mas quando ele diz que não 
devolve troco, me abre margem pra eu colocar dinheiro a mais na maquina ( 25 
mais 3 de 10 ) pagaria o cafe e a maquina não devolveria troco. o que vcs acham 
estou errada ou essa questão seria passível de anulação? ou alteração de 
gabarito para letra c?


1º-10 x52º- 8x 5+ 103º- 6x5+ 2x104º-4x5 + 3x105º-2x5+ 
4x106º-5x107º-25+5x58º-25+3x5+109º-25+5+2x1010º- 2x25


Vanessa Nunes

  

[obm-l] Re: [obm-l] Dúvidas

2012-03-15 Por tôpico Gabriel Merêncio
1 - João atravessa o percurso BT, de 12 km, com velocidade média de 15
km/h, o que significa que ele leva 12/15 * 60 = 48 minutos. Daí, restam 42
minutos para a trajetória CB, a qual João percorre com velocidade média de
10 km/h. Em 42 minutos (ou seja, 42/60 = 7/10 de hora), João percorre uma
distância de 7 km, que é o valor de CB.

Agora note que o ângulo CBA é suplementar ao ângulo CBT, então os senos são
equivalentes. Como o seno é cateto oposto sobre hipotenusa: 0,54 = AC/CB =
AC = 0,54 * 7 = 3,78. Alternativa (A), portanto.

2 - Podemos escrever seno de 30 como:

sen(30º) = sen(15º + 15º) = sen(15º)cos(15º) + sen(15º)cos(15º) =
2sen(15º)cos(15º)

Dividindo esse valor por sen(15º), que é o termo anterior, obtemos
2cos(15º). Alternativa (D).

2012/3/14 Vanessa Nunes de Souza vanessani...@hotmail.com


 Olá, mas uma vez gostaria da ajuda dos colegas em algumas questões de
 concurso.

 1-João se desloca diariamente de sua casa (ponto C) até o trabalho (ponto
 T), passando pelo (ponto B) em trajetórias retilíneas, conforme mostra a
 figura :

 Desenho em anexo

 Considere-se que, num determinado dia, João percorreu: - a distância CB,
 com velocidade média de 10 km/h; - a distância BT = 12 km, com velocidade
 média de 15 km/h;
 - toda essa trajetória em 1h30min.
 Se o seno do ângulo CBT vale 0,54, o segmento CA,
 perpendicular à reta AT, mede, em km:
 (A) 3,78
 (B) 3,29
 (C) 2,56
 (D) 2,14

 2-A sequência (sen15º, sen30º,...) é uma progressão geométrica.
 A razão dessa progressão é igual a:
 (A)
 (B) 2
 (C) sen2°
 (D) 2cos15°

 Agradeço quem puder ajudar.
 Vanessa Nunes



[obm-l] RES: [obm-l] Re: [obm-l] Dúvidas

2012-03-15 Por tôpico Albert Bouskela
Olá!

 

O enunciado da 2ª questão está completamente errado!

 

sin(30°)/sin(15°) = 2cos(15°) = 1,93

 

Supondo que o próx. termo da sequência seja sin(60°): sin(60°)/sin(30°) =
2cos(30°) = 1,73

 

Pior: — A função “sin” é periódica (cresce e decresce periodicamente). Daí
nunca poderá formar uma PG! E mais: — Toda PG é uma função polinomial,
monotonamente crescente para razões maiores do que 1, e decrescente para
razões menores do que 1. A função “sin” não é polinomial!

 

Quem elaborou esta questão???

 

Albert Bouskela

bousk...@msn.com

 

De: owner-ob...@mat.puc-rio.br [mailto:owner-ob...@mat.puc-rio.br] Em nome
de Gabriel Merêncio
Enviada em: 15 de março de 2012 09:47
Para: obm-l@mat.puc-rio.br
Assunto: [obm-l] Re: [obm-l] Dúvidas

 

1 - João atravessa o percurso BT, de 12 km, com velocidade média de 15 km/h,
o que significa que ele leva 12/15 * 60 = 48 minutos. Daí, restam 42 minutos
para a trajetória CB, a qual João percorre com velocidade média de 10 km/h.
Em 42 minutos (ou seja, 42/60 = 7/10 de hora), João percorre uma distância
de 7 km, que é o valor de CB.

 

Agora note que o ângulo CBA é suplementar ao ângulo CBT, então os senos são
equivalentes. Como o seno é cateto oposto sobre hipotenusa: 0,54 = AC/CB =
AC = 0,54 * 7 = 3,78. Alternativa (A), portanto.

 

2 - Podemos escrever seno de 30 como:

 

sen(30º) = sen(15º + 15º) = sen(15º)cos(15º) + sen(15º)cos(15º) =
2sen(15º)cos(15º)

 

Dividindo esse valor por sen(15º), que é o termo anterior, obtemos
2cos(15º). Alternativa (D).

2012/3/14 Vanessa Nunes de Souza vanessani...@hotmail.com


Olá, mas uma vez gostaria da ajuda dos colegas em algumas questões de
concurso.

 

1-João se desloca diariamente de sua casa (ponto C) até o trabalho (ponto
T), passando pelo (ponto B) em trajetórias retilíneas, conforme mostra a
figura :

 

Desenho em anexo

 

Considere-se que, num determinado dia, João percorreu: - a distância CB, com
velocidade média de 10 km/h; - a distância BT = 12 km, com velocidade média
de 15 km/h;

- toda essa trajetória em 1h30min.

Se o seno do ângulo CBT vale 0,54, o segmento CA,

perpendicular à reta AT, mede, em km:

(A) 3,78

(B) 3,29

(C) 2,56

(D) 2,14

 

2-A sequência (sen15º, sen30º,...) é uma progressão geométrica.

A razão dessa progressão é igual a:

(A)

(B) 2

(C) sen2°

(D) 2cos15°

 

Agradeço quem puder ajudar.

Vanessa Nunes

 



[obm-l] Re: [obm-l] RES: [obm-l] Re: [obm-l] Dúvidas

2012-03-15 Por tôpico Ralph Teixeira
Mas quem disse que o proximo termo eh sin(60°)? ;)

Abraco,
Ralph

2012/3/15 Albert Bouskela bousk...@msn.com:
 Olá!



 O enunciado da 2ª questão está completamente errado!



 sin(30°)/sin(15°) = 2cos(15°) = 1,93



 Supondo que o próx. termo da sequência seja sin(60°): sin(60°)/sin(30°) =
 2cos(30°) = 1,73



 Pior: — A função “sin” é periódica (cresce e decresce periodicamente). Daí
 nunca poderá formar uma PG! E mais: — Toda PG é uma função polinomial,
 monotonamente crescente para razões maiores do que 1, e decrescente para
 razões menores do que 1. A função “sin” não é polinomial!



 Quem elaborou esta questão???



 Albert Bouskela

 bousk...@msn.com



 De: owner-ob...@mat.puc-rio.br [mailto:owner-ob...@mat.puc-rio.br] Em nome
 de Gabriel Merêncio
 Enviada em: 15 de março de 2012 09:47
 Para: obm-l@mat.puc-rio.br
 Assunto: [obm-l] Re: [obm-l] Dúvidas



 1 - João atravessa o percurso BT, de 12 km, com velocidade média de 15 km/h,
 o que significa que ele leva 12/15 * 60 = 48 minutos. Daí, restam 42 minutos
 para a trajetória CB, a qual João percorre com velocidade média de 10 km/h.
 Em 42 minutos (ou seja, 42/60 = 7/10 de hora), João percorre uma distância
 de 7 km, que é o valor de CB.



 Agora note que o ângulo CBA é suplementar ao ângulo CBT, então os senos são
 equivalentes. Como o seno é cateto oposto sobre hipotenusa: 0,54 = AC/CB =
 AC = 0,54 * 7 = 3,78. Alternativa (A), portanto.



 2 - Podemos escrever seno de 30 como:



 sen(30º) = sen(15º + 15º) = sen(15º)cos(15º) + sen(15º)cos(15º) =
 2sen(15º)cos(15º)



 Dividindo esse valor por sen(15º), que é o termo anterior, obtemos
 2cos(15º). Alternativa (D).

 2012/3/14 Vanessa Nunes de Souza vanessani...@hotmail.com


 Olá, mas uma vez gostaria da ajuda dos colegas em algumas questões de
 concurso.



 1-João se desloca diariamente de sua casa (ponto C) até o trabalho (ponto
 T), passando pelo (ponto B) em trajetórias retilíneas, conforme mostra a
 figura :



 Desenho em anexo



 Considere-se que, num determinado dia, João percorreu: - a distância CB, com
 velocidade média de 10 km/h; - a distância BT = 12 km, com velocidade média
 de 15 km/h;

 - toda essa trajetória em 1h30min.

 Se o seno do ângulo CBT vale 0,54, o segmento CA,

 perpendicular à reta AT, mede, em km:

 (A) 3,78

 (B) 3,29

 (C) 2,56

 (D) 2,14



 2-A sequência (sen15º, sen30º,...) é uma progressão geométrica.

 A razão dessa progressão é igual a:

 (A)

 (B) 2

 (C) sen2°

 (D) 2cos15°



 Agradeço quem puder ajudar.

 Vanessa Nunes



=
Instruções para entrar na lista, sair da lista e usar a lista em
http://www.mat.puc-rio.br/~obmlistas/obm-l.html
=


[obm-l] RE: [obm-l] Re: [obm-l] Dúvidas

2012-03-15 Por tôpico Vanessa Nunes de Souza

Obrigada Gabriel.
Vanessa Nunes
Date: Thu, 15 Mar 2012 09:47:12 -0300
Subject: [obm-l] Re: [obm-l] Dúvidas
From: gmerencio.san...@gmail.com
To: obm-l@mat.puc-rio.br

1 - João atravessa o percurso BT, de 12 km, com velocidade média de 15 km/h, o 
que significa que ele leva 12/15 * 60 = 48 minutos. Daí, restam 42 minutos para 
a trajetória CB, a qual João percorre com velocidade média de 10 km/h. Em 42 
minutos (ou seja, 42/60 = 7/10 de hora), João percorre uma distância de 7 km, 
que é o valor de CB.

Agora note que o ângulo CBA é suplementar ao ângulo CBT, então os senos são 
equivalentes. Como o seno é cateto oposto sobre hipotenusa: 0,54 = AC/CB = AC 
= 0,54 * 7 = 3,78. Alternativa (A), portanto.

2 - Podemos escrever seno de 30 como:
sen(30º) = sen(15º + 15º) = sen(15º)cos(15º) + sen(15º)cos(15º) = 
2sen(15º)cos(15º)
Dividindo esse valor por sen(15º), que é o termo anterior, obtemos 2cos(15º). 
Alternativa (D).


2012/3/14 Vanessa Nunes de Souza vanessani...@hotmail.com






Olá, mas uma vez gostaria da ajuda dos colegas em algumas questões de concurso.
1-João se desloca diariamente de sua casa (ponto C) até o trabalho (ponto T), 
passando pelo (ponto B) em trajetórias retilíneas, conforme mostra a figura :

Desenho em anexo
Considere-se que, num determinado dia, João percorreu: - a distância CB, com 
velocidade média de 10 km/h; - a distância BT = 12 km, com velocidade média de 
15 km/h;
- toda essa trajetória em 1h30min.Se o seno do ângulo CBT vale 0,54, o segmento 
CA,perpendicular à reta AT, mede, em km:(A) 3,78(B) 3,29(C) 2,56(D) 2,14

2-A sequência (sen15º, sen30º,...) é uma progressão geométrica.A razão dessa 
progressão é igual a:(A)(B) 2(C) sen2°
(D) 2cos15°
Agradeço quem puder ajudar.Vanessa Nunes
  

  

[obm-l] RES: [obm-l] Re: [obm-l] RES: [obm-l] Re: [obm-l] Dúvidas

2012-03-15 Por tôpico Albert Bouskela
Tá bom... o próx. termo é sin(75°) — veja o meu e-mail anterior.

Te dou um doce se você achar o próximo! :-)))

Abraço,
Albert Bouskela
bousk...@msn.com


 -Mensagem original-
 De: owner-ob...@mat.puc-rio.br [mailto:owner-ob...@mat.puc-rio.br] Em nome
de
 Ralph Teixeira
 Enviada em: 15 de março de 2012 11:44
 Para: obm-l@mat.puc-rio.br
 Assunto: [obm-l] Re: [obm-l] RES: [obm-l] Re: [obm-l] Dúvidas
 
 Mas quem disse que o proximo termo eh sin(60°)? ;)
 
 Abraco,
 Ralph
 
 2012/3/15 Albert Bouskela bousk...@msn.com:
  Olá!
 
 
 
  O enunciado da 2ª questão está completamente errado!
 
 
 
  sin(30°)/sin(15°) = 2cos(15°) = 1,93
 
 
 
  Supondo que o próx. termo da sequência seja sin(60°):
  sin(60°)/sin(30°) =
  2cos(30°) = 1,73
 
 
 
  Pior: — A função “sin” é periódica (cresce e decresce periodicamente).
  Daí nunca poderá formar uma PG! E mais: — Toda PG é uma função
  polinomial, monotonamente crescente para razões maiores do que 1, e
  decrescente para razões menores do que 1. A função “sin” não é
polinomial!
 
 
 
  Quem elaborou esta questão???
 
 
 
  Albert Bouskela
 
  bousk...@msn.com
 
 
 
  De: owner-ob...@mat.puc-rio.br [mailto:owner-ob...@mat.puc-rio.br] Em
  nome de Gabriel Merêncio Enviada em: 15 de março de 2012 09:47
  Para: obm-l@mat.puc-rio.br
  Assunto: [obm-l] Re: [obm-l] Dúvidas
 
 
 
  1 - João atravessa o percurso BT, de 12 km, com velocidade média de 15
  km/h, o que significa que ele leva 12/15 * 60 = 48 minutos. Daí,
  restam 42 minutos para a trajetória CB, a qual João percorre com
velocidade
 média de 10 km/h.
  Em 42 minutos (ou seja, 42/60 = 7/10 de hora), João percorre uma
  distância de 7 km, que é o valor de CB.
 
 
 
  Agora note que o ângulo CBA é suplementar ao ângulo CBT, então os
  senos são equivalentes. Como o seno é cateto oposto sobre hipotenusa:
  0,54 = AC/CB = AC = 0,54 * 7 = 3,78. Alternativa (A), portanto.
 
 
 
  2 - Podemos escrever seno de 30 como:
 
 
 
  sen(30º) = sen(15º + 15º) = sen(15º)cos(15º) + sen(15º)cos(15º) =
  2sen(15º)cos(15º)
 
 
 
  Dividindo esse valor por sen(15º), que é o termo anterior, obtemos
  2cos(15º). Alternativa (D).
 
  2012/3/14 Vanessa Nunes de Souza vanessani...@hotmail.com
 
 
  Olá, mas uma vez gostaria da ajuda dos colegas em algumas questões de
  concurso.
 
 
 
  1-João se desloca diariamente de sua casa (ponto C) até o trabalho
  (ponto T), passando pelo (ponto B) em trajetórias retilíneas, conforme
  mostra a figura :
 
 
 
  Desenho em anexo
 
 
 
  Considere-se que, num determinado dia, João percorreu: - a distância
  CB, com velocidade média de 10 km/h; - a distância BT = 12 km, com
  velocidade média de 15 km/h;
 
  - toda essa trajetória em 1h30min.
 
  Se o seno do ângulo CBT vale 0,54, o segmento CA,
 
  perpendicular à reta AT, mede, em km:
 
  (A) 3,78
 
  (B) 3,29
 
  (C) 2,56
 
  (D) 2,14
 
 
 
  2-A sequência (sen15º, sen30º,...) é uma progressão geométrica.
 
  A razão dessa progressão é igual a:
 
  (A)
 
  (B) 2
 
  (C) sen2°
 
  (D) 2cos15°
 
 
 
  Agradeço quem puder ajudar.
 
  Vanessa Nunes
 
 
 
 =
 
 Instruções para entrar na lista, sair da lista e usar a lista em
http://www.mat.puc-
 rio.br/~obmlistas/obm-l.html
 =
 


=
Instruções para entrar na lista, sair da lista e usar a lista em
http://www.mat.puc-rio.br/~obmlistas/obm-l.html
=


[obm-l] Re: [obm-l] RES: [obm-l] Re: [obm-l] RES: [obm-l] Re: [obm-l] Dúvidas

2012-03-15 Por tôpico Bernardo Freitas Paulo da Costa
2012/3/15 Albert Bouskela bousk...@msn.com:
 Tá bom... o próx. termo é sin(75°) — veja o meu e-mail anterior.

 Te dou um doce se você achar o próximo! :-)))
Você sabe que eu gosto de análise, né? Complexa é melhor ainda.

sin(pi/2 + I* arcsinh(1/8 * 1/sin(pi/12)^2) está na PG, assim como
sin(pi/2 + I*arcsin(1/16 * 1/sin(pi/12)^3)), etc, etc.

Ah, e se você acha arcsinh feio (porque o resto é bonitinho, mesmo
o seno de 15° que eu não expandi pra economizar espaço), é só um
logaritmo de uma raiz de uma equação de segundo grau, então é bem
explícito ;)

Observações:
1) a função seno nos complexos tem exatamente o mesmo problema da
exponencial, ou seja, arco seno não é bem definido, da mesma forma que
o logaritmo, afinal de contas, ela continua sendo 2pi periódica!
Assim, não apenas esses valores, mas infinitos mais (como acima) fazem
parte da seqüência. O mais estranho é que começa no eixo real, e
termina numa vertical imaginária sobre pi/2.

2) Aliás, assimptoticamente, os argumentos estarão em PA, porque
afinal de contas sin(pi/2 + iy) = sinh(y) que é quase igual à
exponencial exp(y)/2, ainda mais quando y - infinito, e como os
termos da PG também formam uma função exponencial a*exp(k*n), tudo
fica bonitinho.

Exercício: calcular o erro da falsa PA das ordenadas imaginárias no
resto da seqüência, e encontrar o limite
log(erro_n)/n (ou seja, a velocidade do decaimento, e a dica é que a
ordem é exponencial, o que é de se esperar).

O primeiro que responder esse exercício ganha o doce do Bouskela :).

Abraços reais e imaginários,
-- 
Bernardo Freitas Paulo da Costa

=
Instruções para entrar na lista, sair da lista e usar a lista em
http://www.mat.puc-rio.br/~obmlistas/obm-l.html
=


[obm-l] RES: [obm-l] Re: [obm-l] RES: [obm-l] Re: [obm-l] Dúvidas

2012-03-15 Por tôpico Albert Bouskela
Olá, Ralph!

Jogo a toalha! Você merece o doce! Devo, não nego, pagarei quando puder!

Mas eu posso me justificar (só pra não ficar coberto de vergonha!):

Qdo. vi a questão, vi assim: [ sin(15°), sin(30°)... ] e, não, [ sin(15°),
sin(30°), ... , ].

A diferença é sutil, reconheço! Entretanto, embora correta, a questão me
induziu ao erro — e isto não é correto!

Afinal, qualquer cristão (sou materialista dialético!), ao olhar para [
sin(15°), sin(30°), ... ], enxerga [ sin(15°), sin(30°), sin(60°)... ].
Repare que a questão refere-se a uma PG, então o termo PG ficou martelando
na minha cabeça!

Bem, ao ler isto aí de cima, soa como uma desculpa pra lá de esfarrapada — é
verdade!

Contudo, há um fato incontrastável (palavra bonita!): — eu erraria a
questão! Mas eu tenho todos (todos!) os conhecimentos necessários para
resolvê-la fácil e rapidamente! Mesmo assim eu erraria! Só posso concluir
que questão esteja mal formulada.

Abraço,
Albert Bouskela
bousk...@msn.com


 -Mensagem original-
 De: ralp...@gmail.com [mailto:ralp...@gmail.com] Em nome de Ralph Teixeira
 Enviada em: 15 de março de 2012 13:02
 Para: bousk...@msn.com
 Assunto: Re: [obm-l] Re: [obm-l] RES: [obm-l] Re: [obm-l] Dúvidas
 
 Oi, Bouskela.
 
 Os termos sao
 
 sin15
 sin30
 2sin30cos15
 4sin30(cos15)^2
 8sin30(cos15)^3
 ...
 
 :P
 
 Concordo contigo que fica dificil fazer uma P.A. com um bando de senos
(bom,
 especialmente com os argumentos em P.A. ou P.G.) -- voce demonstrou isso
ali
 em cima. Mas eles colocaram ... no terceiro termo. Ninguem falou que os
termos
 sao um bando de senos, muito menos de senos de arcos em P.A. Eles disseram
 que eh uma P.G., e que os dois primeiros termos sao (raiz(6)-raiz(2))/4 e
1/2, soh
 isso.
 
 Alias, acho que eh o contrario -- o PONTO da questao eh exatamente que nao
se
 faz uma P.G. simplesmente botando P.G. ou P.A. nos argumentos dos senos.
 Note a resposta errada (C) -- se algum aluno achar que sina.sinb =
sin(ab), ele
 marca sin2 como resposta, e acha que a P.G.
 eh sin15, sin30, sin60, sin120, sin240 -- que, como voce disse seria
absurdo.
 
 Em suma, acho que a questao estah corretissima, e ela estah tentando
ensinar
 exatamente o que voce afirma corretamente. Ela concorda contigo!
 
 Abraco,
   Ralph
 
 P.S.: Quero meu doce! ;)
 
 2012/3/15 Albert Bouskela bousk...@msn.com:
  Tá bom... o próx. termo é sin(75°) — veja o meu e-mail anterior.
 
 
 
  Te dou um doce se você achar o próximo! :-)))
 
 
 
  Veja o e-mail que enviei ao Bernardo (está abaixo)
 
 
 
  Abraço,
 
  Albert Bouskela
 
  bousk...@msn.com
 
 
 
  E-mail enviado ao Bernardo:
 
 
 
  Olá!
 
 
 
  O que quis dizer é:
 
 
 
  PG: [ a, a(r), a(r^2), a(r^3) ... a(r^n) ] , um polinômio de grau n
  (estou me referindo a uma PG finita).
 
  sin: [ sin(arg1), sin(arg2), sin(arg3) ... sin(argn) ] , sendo
  argi = argumento de índice i.
 
 
 
  Daí, é necessário (necessário para este problema, é claro!) encontrar
  uma equação recursiva entre argi e argi+1 que satisfaça à:
 
 
 
  sin(argi) = a(r^k) , para i=a...b e k=c...d , a e b inteiros,
  assim como também c e d. Veja que se pode fazer k=1...n sem perda
  de generalidade.
 
 
 
  Não é possível encontrar esta equação recursiva, porque [ sin(arg1),
  sin(arg2), sin(arg3) ... sin(argn) ] não é um polinômio para
  qualquer i (inteiro).
 
 
 
  É claro que poderíamos fazer: [ sin(15°), sin(30°), sin(75°) ] .
  Realmente uma PG! Mas não é este o enunciado do problema!!!
 
 
 
  E mais: — O próximo (4°) termo desta PG é maior do que 1 (igual a 1,87).
  Portanto, já não pode ser expresso como o seno de um ângulo.
 
 
 
  O enunciado do problema (tal como está!) não faz sentido. É mais fácil
  ver isto através dos outros argumentos que apresentei:
 
 
 
  sin(30°)/sin(15°) = 2cos(15°) = 1,93
 
 
 
  Supondo que o próx. termo da sequência seja sin(60°):
  sin(60°)/sin(30°) =
  2cos(30°) = 1,73
 
 
 
  1,93 é diferente de 1,73 (não vou colocar ! para não ficar parecendo
  o fatorial de 1,73).
 
 
 
  Supondo que o próx. termo da sequência seja sin(45°), também não forma
  uma PG!
 
 
 
  A função “sin” é periódica (cresce e decresce periodicamente). Daí
  nunca poderá formar uma PG! Toda PG é monotonamente crescente para
  razões maiores do que 1, e monotonamente decrescente para razões menores
 do que 1.
 
 
 
  Albert Bouskela
 
  bousk...@msn.com


=
Instruções para entrar na lista, sair da lista e usar a lista em
http://www.mat.puc-rio.br/~obmlistas/obm-l.html
=


[obm-l] Re: [obm-l] Re: [obm-l] RE: [obm-l] Re : [obm-l] RE: [obm-l] Dúvidas

2010-03-22 Por tôpico Paulo Barclay Ribeiro

Eduardo e Marcone e demais cilegas da Lista.Muito obrigado pelas orientações 
dadas , e acima de tudo agradeço a consideração de todos vocês.
 
Um abraço
 
Paulobarclay
--- Em dom, 21/3/10, Eduardo Wilner eduardowil...@yahoo.com.br escreveu:


De: Eduardo Wilner eduardowil...@yahoo.com.br
Assunto: [obm-l] Re: [obm-l] RE: [obm-l] Re: [obm-l] RE: [obm-l] Dúvidas
Para: obm-l@mat.puc-rio.br
Data: Domingo, 21 de Março de 2010, 0:25






Esquece... No avançado das horas confundí milhar com centena...Desculpe.

--- Em sex, 19/3/10, marcone augusto araújo borges 
marconeborge...@hotmail.com escreveu:


De: marcone augusto araújo borges marconeborge...@hotmail.com
Assunto: [obm-l] RE: [obm-l] Re: [obm-l] RE: [obm-l] Dúvidas
Para: obm-l@mat.puc-rio.br
Data: Sexta-feira, 19 de Março de 2010, 20:22




De onde sai b-a=4?8001+100a+10b-6633=1008+100b+10a implica 
8001-6633-1008=100b-10b+10a-100a.Dai 360=90*(b-a).Então b-a=4.Sobre a segunda 
pergunta,não entendi.Abraço. 
 


Date: Thu, 18 Mar 2010 18:26:38 -0700
From: eduardowil...@yahoo.com.br
Subject: [obm-l] Re: [obm-l] RE: [obm-l] Dúvidas
To: obm-l@mat.puc-rio.br





De onde sai b - a = 4 ? O problema diz que E mais a diferença entre o 
algarismo das unidades de milhar e do das unidades simples é igual a 7 ... 

 O das unidades deve ser no mínimo 6, assim deveria ser o das unidades menos o 
do milhar igual a 7 ???


--- Em qui, 18/3/10, marcone augusto araújo borges 
marconeborge...@hotmail.com escreveu:


De: marcone augusto araújo borges marconeborge...@hotmail.com
Assunto: [obm-l] RE: [obm-l] Dúvidas
Para: obm-l@mat.puc-rio.br
Data: Quinta-feira, 18 de Março de 2010, 21:18




#yiv1800976672 #yiv1626342901 .ExternalClass #ecxyiv1454970140 .ecxhmmessage P
{padding:0px;}
#yiv1800976672 #yiv1626342901 .ExternalClass #ecxyiv1454970140 .ecxhmmessage
{font-size:10pt;font-family:Verdana;}

Sobre a questão 2,uma das possibilidades é o numero (8ab1),q subtraido de 6633 
dá 8001+100a+10b-6633=(1ba8)=1008+100b+10a.Dai b-a=4.Obtemos assim os números 
8041(8041-6633=1408);8151;8261;8371;8481 e 8591.Ai tem q ver se tem mais 
números da forma 8ab1.Outra possibilidade é o número (9xy2)...depois vejo se 
concluo,mas acho q já é um bom caminho...
Abraço
 


Date: Wed, 17 Mar 2010 10:51:55 -0700
From: paulobarc...@yahoo.com.br
Subject: [obm-l] Dúvidas
To: obm-l@mat.puc-rio.br






Oi Pessoal.
Peço uma orientação para resolver os seguintes problemas:
 
1)Dada uma PG infinita com razão entre 0 e 1 do tipo a_1 , a_2..a_n...
 
Tiram-se dela as PG's igualmente infinitas:
a) a_1, a_3,a_6a_3n.  cuja soma é 8.
b) a_4, a_8, a_12.a_4n cuja soma é quatro quintos.
 
Determine a soma da PG original.
 
Neste problema acho uma razão maior do que 1. Acho que na primeira PG o termo 
a_1 não deveria figurar, por favor me digam se estou com a razão.
 
2) Um número inteiro positivo k possui 4 algarismos.Subtraindo-se dele o número 
6633 o obtem-se um número que é obtido invertendo-se a ordem dos algarismos de 
k.E mais a diferença entre o algarismo das unidades de milhar e do das unidades 
simples é igual a 7.Quantos númros inteiros positivos k existem com essas 
caracteristicas?
 
Desde já agradeço a atenção
 
Grato
 
Paulobarclay
 
 
 


Veja quais são os assuntos do momento no Yahoo! + Buscados: Top 10 - 
Celebridades - Música - Esportes 


Transforme-se em personagens engraçados. Conheça o novo site de I Love 
Messenger. 


Veja quais são os assuntos do momento no Yahoo! + Buscados: Top 10 - 
Celebridades - Música - Esportes 


Quer falar com seus amigos do Messenger sem instalar nada? Clique aqui e veja 
como. 


Veja quais são os assuntos do momento no Yahoo! + Buscados: Top 10 - 
Celebridades - Música - Esportes


  

Veja quais são os assuntos do momento no Yahoo! +Buscados
http://br.maisbuscados.yahoo.com

[obm-l] Re: [obm-l] RE: [obm-l] Re: [obm-l] RE : [obm-l] Dúvidas

2010-03-20 Por tôpico Eduardo Wilner
Esquece... No avançado das horas confundí milhar com centena...Desculpe.

--- Em sex, 19/3/10, marcone augusto araújo borges 
marconeborge...@hotmail.com escreveu:

De: marcone augusto araújo borges marconeborge...@hotmail.com
Assunto: [obm-l] RE: [obm-l] Re: [obm-l] RE: [obm-l] Dúvidas
Para: obm-l@mat.puc-rio.br
Data: Sexta-feira, 19 de Março de 2010, 20:22




De onde sai b-a=4?8001+100a+10b-6633=1008+100b+10a implica 
8001-6633-1008=100b-10b+10a-100a.Dai 360=90*(b-a).Então b-a=4.Sobre a segunda 
pergunta,não entendi.Abraço. 
 


Date: Thu, 18 Mar 2010 18:26:38 -0700
From: eduardowil...@yahoo.com.br
Subject: [obm-l] Re: [obm-l] RE: [obm-l] Dúvidas
To: obm-l@mat.puc-rio.br





De onde sai b - a = 4 ? O problema diz que E mais a diferença entre o 
algarismo das unidades de milhar e do das unidades simples é igual a 7 ... 

 O das unidades deve ser no mínimo 6, assim deveria ser o das unidades menos o 
do milhar igual a 7 ???


--- Em qui, 18/3/10, marcone augusto araújo borges 
marconeborge...@hotmail.com escreveu:


De: marcone augusto araújo borges marconeborge...@hotmail.com
Assunto: [obm-l] RE: [obm-l] Dúvidas
Para: obm-l@mat.puc-rio.br
Data: Quinta-feira, 18 de Março de 2010, 21:18




#yiv1626342901 .ExternalClass #ecxyiv1454970140 .ecxhmmessage P
{padding:0px;}
#yiv1626342901 .ExternalClass #ecxyiv1454970140 .ecxhmmessage
{font-size:10pt;font-family:Verdana;}

Sobre a questão 2,uma das possibilidades é o numero (8ab1),q subtraido de 6633 
dá 8001+100a+10b-6633=(1ba8)=1008+100b+10a.Dai b-a=4.Obtemos assim os números 
8041(8041-6633=1408);8151;8261;8371;8481 e 8591.Ai tem q ver se tem mais 
números da forma 8ab1.Outra possibilidade é o número (9xy2)...depois vejo se 
concluo,mas acho q já é um bom caminho...
Abraço
 


Date: Wed, 17 Mar 2010 10:51:55 -0700
From: paulobarc...@yahoo.com.br
Subject: [obm-l] Dúvidas
To: obm-l@mat.puc-rio.br






Oi Pessoal.
Peço uma orientação para resolver os seguintes problemas:
 
1)Dada uma PG infinita com razão entre 0 e 1 do tipo a_1 , a_2..a_n...
 
Tiram-se dela as PG's igualmente infinitas:
a) a_1, a_3,a_6a_3n.  cuja soma é 8.
b) a_4, a_8, a_12.a_4n cuja soma é quatro quintos.
 
Determine a soma da PG original.
 
Neste problema acho uma razão maior do que 1. Acho que na primeira PG o termo 
a_1 não deveria figurar, por favor me digam se estou com a razão.
 
2) Um número inteiro positivo k possui 4 algarismos.Subtraindo-se dele o número 
6633 o obtem-se um número que é obtido invertendo-se a ordem dos algarismos de 
k.E mais a diferença entre o algarismo das unidades de milhar e do das unidades 
simples é igual a 7.Quantos númros inteiros positivos k existem com essas 
caracteristicas?
 
Desde já agradeço a atenção
 
Grato
 
Paulobarclay
 
 
 


Veja quais são os assuntos do momento no Yahoo! + Buscados: Top 10 - 
Celebridades - Música - Esportes 


Transforme-se em personagens engraçados. Conheça o novo site de I Love 
Messenger. 


Veja quais são os assuntos do momento no Yahoo! + Buscados: Top 10 - 
Celebridades - Música - Esportes 
Quer falar com seus amigos do Messenger sem instalar nada? Clique aqui e veja 
como. 



  

Veja quais são os assuntos do momento no Yahoo! +Buscados
http://br.maisbuscados.yahoo.com

[obm-l] Re: [obm-l] RE: [obm-l] Dúvidas

2010-03-20 Por tôpico Paulo Barclay Ribeiro
Oi Luis , agradeço a sua ajuada.A raiz da equação é 1/2.
A dúvida que estou tendo  é que na primeira sub PG   a_1,a_3,...a_3n . O 
promeiro termo não me parece adequado , pois na hora de calcular a razão dessa 
nova PG : a_3/a_1  difere de a_6 / a_3. Quando Eliminamos o a_1 a razão passa a 
ser 1/10 e a soma fica 8,88.. que não é aquela dada no enunciado.Se você 
considerar o a_1, como está no enunciado,. a soma vai para 888,888Aí parei!
 
Desde ,mais uma vez , agradeço a sua ajuda e de todo pessoal da lista.Fiquem, a 
vontade para me corrigirem , tô aqui pra aprender .
um abraço
 
PauloBarclay
 

--- Em qui, 18/3/10, Luís Lopes qed_te...@hotmail.com escreveu:


De: Luís Lopes qed_te...@hotmail.com
Assunto: [obm-l] RE: [obm-l] Dúvidas
Para: obm-l@mat.puc-rio.br
Data: Quinta-feira, 18 de Março de 2010, 16:44




Sauda,c~oes, 
 
Desculpem pelo envio de mensagens mais ou menos repetidas. 
 
Vamos ver se esta chega com uma resposta somente. 
 
Fiz o sistema (a_2/q)/(1-q^2) = 8 e (a_2q^2)/(1-q^4) = 4/5. 
 
Resolvendo encontro 

10q^3 = 1 + q^2 
 
E parei aqui. q = ? 
 
[]'s 
Luis 
 


 


Date: Wed, 17 Mar 2010 10:51:55 -0700
From: paulobarc...@yahoo.com.br
Subject: [obm-l] Dúvidas
To: obm-l@mat.puc-rio.br






Oi Pessoal.
Peço uma orientação para resolver os seguintes problemas:
 
1)Dada uma PG infinita com razão entre 0 e 1 do tipo a_1 , a_2..a_n...
 
Tiram-se dela as PG's igualmente infinitas:
a) a_1, a_3,a_6a_3n.  cuja soma é 8.
b) a_4, a_8, a_12.a_4n cuja soma é quatro quintos.
 
Determine a soma da PG original.
 
Neste problema acho uma razão maior do que 1. Acho que na primeira PG o termo 
a_1 não deveria figurar, por favor me digam se estou com a razão.
 
2) Um número inteiro positivo k possui 4 algarismos.Subtraindo-se dele o número 
6633 o obtem-se um número que é obtido invertendo-se a ordem dos algarismos de 
k.E mais a diferença entre o algarismo das unidades de milhar e do das unidades 
simples é igual a 7.Quantos númros inteiros positivos k existem com essas 
caracteristicas?
 
Desde já agradeço a atenção
 
Grato
 
Paulobarclay
 
 
 


Veja quais são os assuntos do momento no Yahoo! + Buscados: Top 10 - 
Celebridades - Música - Esportes 


Coloque sua foto num tema anos 60, 70 e 80. Conheça o novo site de I Love 
Messenger. 


  

Veja quais são os assuntos do momento no Yahoo! +Buscados
http://br.maisbuscados.yahoo.com

[obm-l] RE: [obm-l] Dúvidas

2010-03-18 Por tôpico Luís Lopes

Sauda,c~oes, 

 

Desculpem pelo envio de mensagens mais ou menos repetidas. 

 

Vamos ver se esta chega com uma resposta somente. 

 

Fiz o sistema (a_2/q)/(1-q^2) = 8 e (a_2q^2)/(1-q^4) = 4/5. 

 

Resolvendo encontro 


10q^3 = 1 + q^2 

 

E parei aqui. q = ? 

 

[]'s 

Luis 

 



 


Date: Wed, 17 Mar 2010 10:51:55 -0700
From: paulobarc...@yahoo.com.br
Subject: [obm-l] Dúvidas
To: obm-l@mat.puc-rio.br






Oi Pessoal.
Peço uma orientação para resolver os seguintes problemas:
 
1)Dada uma PG infinita com razão entre 0 e 1 do tipo a_1 , a_2..a_n...
 
Tiram-se dela as PG's igualmente infinitas:
a) a_1, a_3,a_6a_3n.  cuja soma é 8.
b) a_4, a_8, a_12.a_4n cuja soma é quatro quintos.
 
Determine a soma da PG original.
 
Neste problema acho uma razão maior do que 1. Acho que na primeira PG o termo 
a_1 não deveria figurar, por favor me digam se estou com a razão.
 
2) Um número inteiro positivo k possui 4 algarismos.Subtraindo-se dele o número 
6633 o obtem-se um número que é obtido invertendo-se a ordem dos algarismos de 
k.E mais a diferença entre o algarismo das unidades de milhar e do das unidades 
simples é igual a 7.Quantos númros inteiros positivos k existem com essas 
caracteristicas?
 
Desde já agradeço a atenção
 
Grato
 
Paulobarclay
 
 
 


Veja quais são os assuntos do momento no Yahoo! + Buscados: Top 10 - 
Celebridades - Música - Esportes 
_
Navegue sem medo com o Internet Explorer 8. Clique aqui para instalar 
gratuitamente.
http://go.microsoft.com/?linkid=9707132

[obm-l] RE: [obm-l] Dúvidas

2010-03-18 Por tôpico marcone augusto araújo borges

Sobre a questão 2,uma das possibilidades é o numero (8ab1),q subtraido de 6633 
dá 8001+100a+10b-6633=(1ba8)=1008+100b+10a.Dai b-a=4.Obtemos assim os números 
8041(8041-6633=1408);8151;8261;8371;8481 e 8591.Ai tem q ver se tem mais 
números da forma 8ab1.Outra possibilidade é o número (9xy2)...depois vejo se 
concluo,mas acho q já é um bom caminho...

Abraço
 


Date: Wed, 17 Mar 2010 10:51:55 -0700
From: paulobarc...@yahoo.com.br
Subject: [obm-l] Dúvidas
To: obm-l@mat.puc-rio.br






Oi Pessoal.
Peço uma orientação para resolver os seguintes problemas:
 
1)Dada uma PG infinita com razão entre 0 e 1 do tipo a_1 , a_2..a_n...
 
Tiram-se dela as PG's igualmente infinitas:
a) a_1, a_3,a_6a_3n.  cuja soma é 8.
b) a_4, a_8, a_12.a_4n cuja soma é quatro quintos.
 
Determine a soma da PG original.
 
Neste problema acho uma razão maior do que 1. Acho que na primeira PG o termo 
a_1 não deveria figurar, por favor me digam se estou com a razão.
 
2) Um número inteiro positivo k possui 4 algarismos.Subtraindo-se dele o número 
6633 o obtem-se um número que é obtido invertendo-se a ordem dos algarismos de 
k.E mais a diferença entre o algarismo das unidades de milhar e do das unidades 
simples é igual a 7.Quantos númros inteiros positivos k existem com essas 
caracteristicas?
 
Desde já agradeço a atenção
 
Grato
 
Paulobarclay
 
 
 


Veja quais são os assuntos do momento no Yahoo! + Buscados: Top 10 - 
Celebridades - Música - Esportes 
_
Com o Internet Explorer 8 você fica mais protegido contra ameaças da web. Saiba 
mais.
http://go.microsoft.com/?linkid=9707132

[obm-l] Re: [obm-l] Dúvidas

2010-03-18 Por tôpico Eduardo Wilner
Realmente, para ser uma PG, a_1 não pode aparecer. Além disso, iniciando essa 
progressão com a_3,  a unica solução positiva para a razão seria 1 o que é 
imcompatível... .

--- Em qua, 17/3/10, Paulo Barclay Ribeiro paulobarc...@yahoo.com.br escreveu:

De: Paulo Barclay Ribeiro paulobarc...@yahoo.com.br
Assunto: [obm-l] Dúvidas
Para: obm-l@mat.puc-rio.br
Data: Quarta-feira, 17 de Março de 2010, 14:51

Oi Pessoal.
Peço uma orientação para resolver os seguintes problemas:
 
1)Dada uma PG infinita com razão entre 0 e 1 do tipo a_1 , a_2..a_n...
 
Tiram-se dela as PG's igualmente infinitas:
a) a_1, a_3,a_6a_3n.  cuja soma é 8.
b) a_4, a_8, a_12.a_4n cuja soma é quatro quintos.
 
Determine a soma da PG original.
 
Neste problema acho uma razão maior do que 1. Acho que na primeira PG o termo 
a_1 não deveria figurar, por favor me digam se estou com a razão.
 
2) Um número inteiro positivo k possui 4 algarismos.Subtraindo-se dele o número 
6633 o obtem-se um número que é obtido invertendo-se a ordem dos algarismos de 
k.E mais a diferença entre o algarismo das unidades de milhar e do das unidades 
simples é igual a 7.Quantos númros inteiros positivos k existem com essas 
caracteristicas?
 
Desde já agradeço a atenção
 
Grato
 
Paulobarclay
 
 
 



  Veja quais são os assuntos do momento no Yahoo! + Buscados: Top 10 - 
Celebridades - Música - Esportes


  

Veja quais são os assuntos do momento no Yahoo! +Buscados
http://br.maisbuscados.yahoo.com

[obm-l] Re: [obm-l] RE: [obm-l] Dúvidas

2010-03-18 Por tôpico Eduardo Wilner
De onde sai b - a = 4 ? O problema diz que E mais a diferença entre o 
algarismo das unidades de milhar e do das unidades simples é igual a 7 ... 

 O das unidades deve ser no mínimo 6, assim deveria ser o das unidades menos o 
do milhar igual a 7 ???


--- Em qui, 18/3/10, marcone augusto araújo borges 
marconeborge...@hotmail.com escreveu:

De: marcone augusto araújo borges marconeborge...@hotmail.com
Assunto: [obm-l] RE: [obm-l] Dúvidas
Para: obm-l@mat.puc-rio.br
Data: Quinta-feira, 18 de Março de 2010, 21:18




Sobre a questão 2,uma das possibilidades é o numero (8ab1),q subtraido de 6633 
dá 8001+100a+10b-6633=(1ba8)=1008+100b+10a.Dai b-a=4.Obtemos assim os números 
8041(8041-6633=1408);8151;8261;8371;8481 e 8591.Ai tem q ver se tem mais 
números da forma 8ab1.Outra possibilidade é o número (9xy2)...depois vejo se 
concluo,mas acho q já é um bom caminho...

Abraço
 


Date: Wed, 17 Mar 2010 10:51:55 -0700
From: paulobarc...@yahoo.com.br
Subject: [obm-l] Dúvidas
To: obm-l@mat.puc-rio.br






Oi Pessoal.
Peço uma orientação para resolver os seguintes problemas:
 
1)Dada uma PG infinita com razão entre 0 e 1 do tipo a_1 , a_2..a_n...
 
Tiram-se dela as PG's igualmente infinitas:
a) a_1, a_3,a_6a_3n.  cuja soma é 8.
b) a_4, a_8, a_12.a_4n cuja soma é quatro quintos.
 
Determine a soma da PG original.
 
Neste problema acho uma razão maior do que 1. Acho que na primeira PG o termo 
a_1 não deveria figurar, por favor me digam se estou com a razão.
 
2) Um número inteiro positivo k possui 4 algarismos.Subtraindo-se dele o número 
6633 o obtem-se um número que é obtido invertendo-se a ordem dos algarismos de 
k.E mais a diferença entre o algarismo das unidades de milhar e do das unidades 
simples é igual a 7.Quantos númros inteiros positivos k existem com essas 
caracteristicas?
 
Desde já agradeço a atenção
 
Grato
 
Paulobarclay
 
 
 


Veja quais são os assuntos do momento no Yahoo! + Buscados: Top 10 - 
Celebridades - Música - Esportes 
Transforme-se em personagens engraçados. Conheça o novo site de I Love 
Messenger. 



  

Veja quais são os assuntos do momento no Yahoo! +Buscados
http://br.maisbuscados.yahoo.com

[obm-l] Re: [obm-l] Dúvidas - Inequações Modulares

2010-03-05 Por tôpico Bruno França dos Reis
Se vc tem dúvida nesse tipo de exercício, talvez o melhor a fazer seja
entender de forma visual exatamente o que ele diz. Interprete-o.

Desenhe num papel um par de eixos, aí coloque uns pontinhos 1, 2, 3, ... em
cada eixo. Trace a reta de equação 2x - 5, depois transforme-a na função |2x
- 5| (como?). Faça o mesmo para a reta x + 3: desenhe-a, e depois
transforme-a na função |x + 3|.

Depois olhe para o seu desenho e pense no que o exercicio pede: quais sao os
valores de x para os quais o desenho do |2x - 5| fica embaixo do desenho do
|x+3| ?


Bruno
--
Bruno FRANÇA DOS REIS

msn: brunoreis...@hotmail.com
skype: brunoreis666
tel: +55 11 9961-7732

http://brunoreis.com
http://brunoreis.com/tech (en)
http://brunoreis.com/blog (pt)

GPG Key: http://brunoreis.com/bruno-public.key

e^(pi*i)+1=0


2010/3/5 Emanuel Valente emanuelvale...@gmail.com

 Pessoal, estou com dÚvidas no seguinte exercício. Gostaria da ajuda de
 vocÊs:

 |2x - 5|  |x+3|

 Valeu!

 --
 Emanuel

 =
 Instruįões para entrar na lista, sair da lista e usar a lista em
 http://www.mat.puc-rio.br/~obmlistas/obm-l.htmlhttp://www.mat.puc-rio.br/%7Eobmlistas/obm-l.html
 =



[obm-l] Re: [obm-l] Dúvidas combinatórias.

2009-09-24 Por tôpico Lucas Prado Melo
2009/9/23 Lucas Colucci lucascolu...@hotmail.com

  Olá membros da lista, gostaria de uma ajuda ajuda no seguinte problema:

 Os inteiros positivos 1, 2, ..., n são colocados nos vértices de um
 n-ágono. Cada vértice é pintado de:

 *Vermelho, se ambos os números nos vértices vizinhos são maiores do que o
 número neste vértice;
 *Azul, se ambos os números nos vértices vizinhos são menores do que o
 número neste vértice;
 *Branco, se nenhuma das duas condições acima for satisfeita.
 Prove que o número de vértices vermelhos é igual ao número de vértices
 azuis.

 Também gostaria de algum material bom sobre contagem dupla, não
 necessariamente em português, caso alguém conhecesse.


Imagine que saiam flechas dos vértices menores para seus vizinhos maiores
(em vez de uma reta).
Para cada permutação, todo lado possuirá uma flecha correspondente (não
existem dois números iguais).

É possível fazer uma indução. A base é a disposição de flechas num mesmo
sentido (sentido horário por exemplo), nesta situação o número de vértices
de onde partem duas flechas (vértices azuis) é igual ao número de vértices
para onde chegam duas flechas (vértices vermelhos), note que esta base não
representa nenhuma disposição possível de se chegar com os dados do
problema:
a - b - c - d - ... - z - a

O passo de indução seria mudar uma única flecha de sentido e ver que os
números de vértices azuis e vermelhos continuam iguais.


[obm-l] Re: [obm-l] Dúvidas

2009-07-05 Por tôpico Frederico Gomes Elihimas
1.Os termos são da forma Ax^a.y^b.w^c.z^d de forma que a+b+c+d=20 pois o
grau dos monômios desta forma é sempre 20.
Logo a resposta é o número de soluções naturais desta equação linear que é
Cn+(k-1),k-1=C23,3=23.22.21/3!=1771 termos

4. triangulo de pascal tem exatamente como soma somatorio{Cn,p} (com
p=0,1,2,..n) =(1+1)^n=2^n pois os termos seriam do binomio (x+a)^n e pondo
x=1=a teriamos apenas coeficientes nos termos.

3. Teríamos a+b+c+d+e+f+g=7 onde a,b,c,d,e,f,g=0(apagada) ou 1(acesa)
possibilidades: 7 acesas C7,7=1
   6 acesas C7,6=...
   5 acesas C7,5
   4 acesas C7,4
   3 acesas C7,3
   2 acesas C7,2
   1 acesa C7,1
exceto 0 acesas C7,0=1
seria então (1+1)^7 - C7,0 = 2^7 -1=127 maneiras diferentes.
i.e somatorio de todos os coeficientes da oitava(n=7) linha do triangulo de
pascal menos o primeiro que é 1

Linhas:
1 primeira(n=0)
11 segunda(n=1)
121 terceira(n=2)
1331 quarta(n=3)

ainda 1. Se fosse (x + y + w + z)^2=x^2+y^2+z^2+w^2+2xy+2xz+2xw+2yw+2yz+2wz
total C5,3=10 termos (só para ilustrar)

2009/7/5 Vinícius pvni...@gmail.com

 1. Quantos termos possui o desenvolvimento de (x + y + w + z)^20? 3. Em uma
 sala há 7 lâmpadas. De quantos modos esta sala pode ser iluminada? 4. Prove,
 utilizando argumento combinatório, que a soma dos números da nésima linha do
 triângulo de Pascal é 2^n.


[obm-l] RE: [obm-l] Dúvidas

2009-07-05 Por tôpico Lucas Colucci

Um jeito combinatório para o problema 4:

Você tem n espaços e quer preenchê-los com 0's ou 1's. (um número por espaço)
Vamos contar de duas formas diferentes o número de maneiras possíveis.
Primeiramente, o mais óbvio seria considerar cada um dos n espaços. Como em 
cada um há duas opções de número (0 ou 1), há 2^n formas.

O segundo jeito de contar é fazendo combinações de k 0's nos n espaços (que 
conta separadamente os casos com 0, 1, 2, ..., n zeros), com k variando de 0 a 
n, que é:
C(n,0)+C(n, 1)+...+C(n,n)

Como as duas contagens são equivalentes, segue que
C(n,0)+C(n,1)+...+C(n,n)=2^n, sendo que os termos do somatório do primeiro 
membro são exatamente os coeficientes da n-ésima linha do triângulo de Pascal.

Lucas Colucci.


Date: Sun, 5 Jul 2009 15:40:53 -0300
Subject: [obm-l] Dúvidas
From: pvni...@gmail.com
To: obm-l@mat.puc-rio.br

1. Quantos termos possui o desenvolvimento de (x + y + w + z)^20?
3. Em uma sala há 7 lâmpadas. De quantos modos esta sala pode ser iluminada?
4. Prove, utilizando argumento combinatório, que a soma dos números da nésima
linha do triângulo de Pascal é 2^n.
_
Descubra todas as novidades do novo Internet Explorer 8
http://brasil.microsoft.com.br/IE8/mergulhe/?utm_source=MSN%3BHotmailutm_medium=Taglineutm_campaign=IE8

Re: [obm-l] Re: [obm-l] RE: [obm-l] DÚVIDAS INGÊNUAS!

2007-04-26 Por tôpico Johann Peter Gustav Lejeune Dirichlet

bem, distorcendo o enunciado, está escrito Sem fazer A multiplicação e não
sem fazer multiplicações.

Então qualquer solução que não seja 7583*999=7575417 é válida, como por
exemplo a sua aí em cima ou algo do tipo
7583*900+7583*90+7583*9=6824700+682470+68247=7575417 ou qualquer coisa que
fosse uma equivalência :P


Em 24/04/07, Bené [EMAIL PROTECTED] escreveu:


999 = 1000 - 1. Portanto, 7583*999 = 7583000 - 7583.
Benedito

- Original Message -
From: Filipe de Carvalho Hasché [EMAIL PROTECTED]
To: obm-l@mat.puc-rio.br
Sent: Tuesday, April 24, 2007 1:44 PM
Subject: [obm-l] RE: [obm-l] DÚVIDAS INGÊNUAS!



Como posso achar o produto de 7583*999 sem fazer a multiplicação


 ===
 --- eu pensei em fazer 7583*(1000-1) e aplicar a distributiva.
 mas não pode fazer multiplicação...

 1ª dúvida: vale colocar 3 zerinhos à direita em vez de multiplicar por
 mil?
 2ª dúvida: posso multiplicar 7583 por 1? ou isso tb é considerado uma
 multiplicação?
 


Por que chamamos eixo dos x ao eixo horizontal e dos y ao eixo vertical?


 

 Engraçado... hj mesmo estava dando aula pro 1° ano do E.M. e propus o
 seguinte exercício:

 Nas sentença: - x + 2.y - 3 = 0 , identifique:
 --- a variável independente;
 --- a variável dependente;
 --- o coeficiente angular;
 --- o coeficiente linear.

 Obviamente, todos os alunos colocaram q a variável independente era x.
 Taí uma boa hora pra querbrar esse tipo de paradigma tendencioso.

 E se a sentença fosse: - z + 2.w - 3 = 0 ??

 Então é bom deixar claro para o nosso aluno q há paradigmas a serem
 quebrados.
 A variável y também poderia ser adotada como a variável independente.
 E isso enceja no caso em q o eixo horizontal seria a reta dos valores
de
 y.

 Além do mais (já fora desse exemplo), podemos ter um sistema
NÃO-ORTOGONAL
 de coordenadas cartesianas. E aí, se tiver eixo deitado, não haveria
eixo
 em pé.

 Bem... isso tudo deve ser considerado dependendo do estáigo de
aprendizado
 do nosso aluno.
 Aprendizado não é uma seqüência linear de conhecimentos adquiridos.
 Muitas vezes precisamos passar por cima de conceitos formais afim de
 desenvolver uma melhor compreensão de certos asuntos.
 Tão logo possível, podemos retomar esse conceito formal para reaprender
 corretamente aquele assunto.

 Esse assunto dá muito pano pra manga.
 O importante é que devemos ser vigilantes em nossas práticas de ensino.

 Abraços,
 FC.

 _
 MSN Busca: fácil, rápido, direto ao ponto.  http://search.msn.com.br


=
 Instruções para entrar na lista, sair da lista e usar a lista em
 http://www.mat.puc-rio.br/~nicolau/olimp/obm-l.html

=


 --
 No virus found in this incoming message.
 Checked by AVG Free Edition. Version: 7.5.463 / Virus Database:
 269.5.10/774 - Release Date: 23/4/2007 17:26



=
Instruções para entrar na lista, sair da lista e usar a lista em
http://www.mat.puc-rio.br/~nicolau/olimp/obm-l.html
=





--
Ideas are bulletproof.

V


[obm-l] RE: [obm-l] DÚVIDAS INGÊNUAS!

2007-04-24 Por tôpico Filipe de Carvalho Hasché



Como posso achar o produto de 7583*999 sem fazer a multiplicação



===
--- eu pensei em fazer 7583*(1000-1) e aplicar a distributiva.
mas não pode fazer multiplicação...

1ª dúvida: vale colocar 3 zerinhos à direita em vez de multiplicar por 
mil?
2ª dúvida: posso multiplicar 7583 por 1? ou isso tb é considerado uma 
multiplicação?





Por que chamamos eixo dos x ao eixo horizontal e dos y ao eixo vertical?





Engraçado... hj mesmo estava dando aula pro 1° ano do E.M. e propus o 
seguinte exercício:


Nas sentença: - x + 2.y - 3 = 0 , identifique:
--- a variável independente;
--- a variável dependente;
--- o coeficiente angular;
--- o coeficiente linear.

Obviamente, todos os alunos colocaram q a variável independente era x.
Taí uma boa hora pra querbrar esse tipo de paradigma tendencioso.

E se a sentença fosse: - z + 2.w - 3 = 0 ??

Então é bom deixar claro para o nosso aluno q há paradigmas a serem 
quebrados.

A variável y também poderia ser adotada como a variável independente.
E isso enceja no caso em q o eixo horizontal seria a reta dos valores de 
y.


Além do mais (já fora desse exemplo), podemos ter um sistema NÃO-ORTOGONAL 
de coordenadas cartesianas. E aí, se tiver eixo deitado, não haveria eixo 
em pé.


Bem... isso tudo deve ser considerado dependendo do estáigo de aprendizado 
do nosso aluno.

Aprendizado não é uma seqüência linear de conhecimentos adquiridos.
Muitas vezes precisamos passar por cima de conceitos formais afim de 
desenvolver uma melhor compreensão de certos asuntos.
Tão logo possível, podemos retomar esse conceito formal para reaprender 
corretamente aquele assunto.


Esse assunto dá muito pano pra manga.
O importante é que devemos ser vigilantes em nossas práticas de ensino.

Abraços,
FC.

_
MSN Busca: fácil, rápido, direto ao ponto.  http://search.msn.com.br

=
Instruções para entrar na lista, sair da lista e usar a lista em
http://www.mat.puc-rio.br/~nicolau/olimp/obm-l.html
=


[obm-l] Re: [obm-l] RE: [obm-l] DÚVIDAS INGÊNUAS!

2007-04-24 Por tôpico Bené

999 = 1000 - 1. Portanto, 7583*999 = 7583000 - 7583.
Benedito

- Original Message - 
From: Filipe de Carvalho Hasché [EMAIL PROTECTED]

To: obm-l@mat.puc-rio.br
Sent: Tuesday, April 24, 2007 1:44 PM
Subject: [obm-l] RE: [obm-l] DÚVIDAS INGÊNUAS!





Como posso achar o produto de 7583*999 sem fazer a multiplicação



===
--- eu pensei em fazer 7583*(1000-1) e aplicar a distributiva.
mas não pode fazer multiplicação...

1ª dúvida: vale colocar 3 zerinhos à direita em vez de multiplicar por 
mil?
2ª dúvida: posso multiplicar 7583 por 1? ou isso tb é considerado uma 
multiplicação?





Por que chamamos eixo dos x ao eixo horizontal e dos y ao eixo vertical?





Engraçado... hj mesmo estava dando aula pro 1° ano do E.M. e propus o 
seguinte exercício:


Nas sentença: - x + 2.y - 3 = 0 , identifique:
--- a variável independente;
--- a variável dependente;
--- o coeficiente angular;
--- o coeficiente linear.

Obviamente, todos os alunos colocaram q a variável independente era x.
Taí uma boa hora pra querbrar esse tipo de paradigma tendencioso.

E se a sentença fosse: - z + 2.w - 3 = 0 ??

Então é bom deixar claro para o nosso aluno q há paradigmas a serem 
quebrados.

A variável y também poderia ser adotada como a variável independente.
E isso enceja no caso em q o eixo horizontal seria a reta dos valores de 
y.


Além do mais (já fora desse exemplo), podemos ter um sistema NÃO-ORTOGONAL 
de coordenadas cartesianas. E aí, se tiver eixo deitado, não haveria eixo 
em pé.


Bem... isso tudo deve ser considerado dependendo do estáigo de aprendizado 
do nosso aluno.

Aprendizado não é uma seqüência linear de conhecimentos adquiridos.
Muitas vezes precisamos passar por cima de conceitos formais afim de 
desenvolver uma melhor compreensão de certos asuntos.
Tão logo possível, podemos retomar esse conceito formal para reaprender 
corretamente aquele assunto.


Esse assunto dá muito pano pra manga.
O importante é que devemos ser vigilantes em nossas práticas de ensino.

Abraços,
FC.

_
MSN Busca: fácil, rápido, direto ao ponto.  http://search.msn.com.br

=
Instruções para entrar na lista, sair da lista e usar a lista em
http://www.mat.puc-rio.br/~nicolau/olimp/obm-l.html
=


--
No virus found in this incoming message.
Checked by AVG Free Edition. Version: 7.5.463 / Virus Database: 
269.5.10/774 - Release Date: 23/4/2007 17:26





=
Instruções para entrar na lista, sair da lista e usar a lista em
http://www.mat.puc-rio.br/~nicolau/olimp/obm-l.html
=


[obm-l] Re: [obm-l] Dúvidas de matrizes... ainda não entendi

2005-07-23 Por tôpico admath
Muito obrigado, Bruno. A 2ª já entendi.
Quanto a primeira questão consegui resolvê-la da seuinte maneira:
A^2 = 0
A^2 - I^2 = 0 - I^2
(A+I)(A-I) = -I x(-1)
-(A+I)(A-I) = I
(-A-I)(A-I) = I
Logo, (-A-I) é a inversa.
Acho que não está certa...não sei se isso vale pra matrizes: A^2 - I^2 = (A+I)(A-I) 
Alguém pode me ajudar?
Obrigado
--O que vc não entendeu?2) Qual é a definição de matriz inversa? Se B é inversa de A, então B é tal que AB = BA = I, certo?Muito bem. Qual é a definição de multiplicação de matrizes?Se A é m x n, e B é p x q (parafraseando o Shine), o produto AB, por definição, é uma matriz C mxq, que só está definido se n = p (também por definição). Simplesmente aceite as definições. Se vc não entender alguma, pergunte. (ou vá a algum livro... o do Iezzi, Fundamentos de Matemática Elementar, é legal)Numa linguagem bem "chula', devemos ter que o segundo número do tamanho da primeira matriz deve ser igual ao primeiro número do tamanho da segunda matriz.OU SEJA: para que o produto C = AB possa existir, devemos ter n = p, POR DEFINIÇÃO.Queremos então saber condições para que A seja invertível. SE A é invertível, ENTÃO existe B tal que AB = BA = I. De AB = I, tiramos que n = p (*), para que AB esteja definido. De BA = I, tiramos que m = q (**). Ainda pela definição de produto, AB é uma matriz de tamanho mxq, e BA é uma matriz de tamanho nxp. De AB = BA, por igualdade de matrizes (i.e., por definição de igualdade de matrizes), devemos ter que os tamanhos de AB e BA são iguais. Logo m = n, p = q (***).De (*), (**) e (***) (não precisa de tudo isso, mas enfim...), tiramos que m=n=p=q, ou seja, as matrizes A e B são quadradas, de mesmo tamanho.Quanto ao 1: não vejo uma solução mais simples que a do Shine. O que tem de muito avançada? Aí a gente pode tentar explicar melhor.AbraçoBruno
 On 7/22/05, admath [EMAIL PROTECTED] wrote:

1) Seja A uma matriz nilpotente nxn, mostre que A -In é inversível e obtenha sua inversa.
 Gostaria de saber como resolvo este tipo de questão organizadamente, separando a hipótese a tese, essas coisas. 

- Ainda não entendi as resoluções deste exercício. São muito avançadas pra mim. Não tem um jeito mais fácil?

2) A matriz inversa é A-1, onde A-1.A = A.A-1=I
 Por que preciso garantir a matriz A sendo nxn?
- Também ainda não entendi o porquê de ser nxn

Obrigado.-- Bruno França dos Reisemail: bfreis - gmail.comgpg-key: http://planeta.terra.com.br/informatica/brunoreis/brunoreis.key icq: 12626000e^(pi*i)+1=0


[obm-l] RE: [obm-l] DÚVIDAS!

2004-11-12 Por tôpico Rogerio Ponce
Olá Jorge!
O problema termina com a frase não receberei a nota de 10 reais !
Se o avô não der nota alguma, a frase seria verdadeira, causando uma 
contradição.
Se o avor der a nota de 10 reais, a frase seria falsa, causando uma 
contradição.
A única alternativa para o avô é entregar a nota de 100 reais (maximizando o 
ganho do neto!).

O trecho problemático do contra-exemplo segue abaixo:
Vovô não vai me dar a nota de cem reais. Vejam o que acontece. A frase
não pode ser falsa. Se o fosse, Pedro não poderia receber nada e a 
afirmação
passava a ser verdadeira - uma contradição. Contudo, a frase pode 
perfeitamente
ser verdadeira. Basta que o neto receba a nota de cem reais. O avô não tem 
outra
alternativa senão dar-lhe a nota de maior valor.
Está errado, pois a alternativa que torna a frase verdadeira é o avô dar a 
nota de 10 reais!

Abraços,
Rogério.

From: jorgeluis
Meus Amigos! Me ajudem a encontrar o erro neste contra-exemplo abaixo. 
Grato!

Pedro, meu neto, gosta muito de quebra-cabeças e problemas outros de 
desafios.
No dia do seu aniversário mandei chamá-lo e, para testá-lo em lógica
matemática, coloquei em cima da mesa uma nota de 10 reais e outra de 100 
reais.
Aqui tens meu presente de aniversário. Se fizeres uma declaração 
verdadeira
dou-te uma das notas, mas se mentires não recebes nada. O pedro, após 
alguns
momentos de reflexão, concluiu que é muito fácil ganhar uma das notas. Mas 
o
legal é ter certeza de receber a nota mais valiosa. Que frase deve dizer o
Pedro para garantir que o avô lhe dará a nota de 100 reais?

Resolução: É claro que Pedro facilmente ganharia uma das notas - bastava 
dizer
uma frase verdadeira, tipo não receberei a nota de 10 reais.

Mas ele pode
realmente, ir mais longe e obrigar o avô, mesmo contra a vontade deste, a
dar-lhe a nota de maior valor. Para isso terá de dizer uma frase
obrigatoriamente verdadeira mas que não dê alternativa ao avô. Vejamos 
algumas
hipóteses:

1) Pedro afirma, por exemplo: - Vovô vai me dar a nota de cem reais. Neste 
caso,
o avô tem duas alternativas: ou lhe diz que a frase é falsa e a prova é que 
ele
não vai receber nada; ou é de muito bom coração e considera a frase 
verdadeira,
dando-lhe os 100 reais. Pedro fica dependendo da boa vontade do avô.

2) O raciocínio é idêntico se ele afirma: - Vou receber a nota de cem, só 
lhe
podia dar 1 real.

3) Outra frase, poderia ser: - Vou ganhar as duas notas. Aqui a situação 
piora
muito. Esta frase nunca pode ser verdadeira porque o avô tinha dito que lhe
dava uma das notas e nunca duas. Neste caso Pedro nada receberia. Esgotadas 
as
frases em que Pedro diz que o avô lhe dará, é preciso analisar agora as
afirmações sobre o que o avô não lhe dará. Então facilmente se chegará à 
frase
solução: Vovô não vai me dar a nota de cem reais. Vejam o que acontece. A 
frase
não pode ser falsa. Se o fosse, Pedro não poderia receber nada e a 
afirmação
passava a ser verdadeira - uma contradição. Contudo, a frase pode 
perfeitamente
ser verdadeira. Basta que o neto receba a nota de cem reais. O avô não tem 
outra
alternativa senão dar-lhe a nota de maior valor.

OBS: Encontrada a solução, podemos ver que Pedro poderia, se quisesse 
deixar o
avô sem saber o que fazer. Bastava-lhe ter dito: - Vovô não me vai dar nota
nenhuma. Esta frase não pode ser verdadeira. Se o fosse, o neto receberia 
uma
das notas e haveria contradição. Também não pode ser falsa. Se o fosse, o 
avô
não lhe poderia dar nada e a afirmação passava a verdadeira. Nova 
contradição.

Um abraço à todos!

__
WebMail UNIFOR - http://www.unifor.br.
=
Instruções para entrar na lista, sair da lista e usar a lista em
http://www.mat.puc-rio.br/~nicolau/olimp/obm-l.html
=
_
MSN Hotmail, o maior webmail do Brasil.  http://www.hotmail.com
=
Instruções para entrar na lista, sair da lista e usar a lista em
http://www.mat.puc-rio.br/~nicolau/olimp/obm-l.html
=


[obm-l] Re: [obm-l] Re: [obm-l] dúvidas de limite e problema legal 6x6

2004-11-09 Por tôpico Artur Costa Steiner
De fato, se o intervalo fechado contiver um aberto que contenha o ponto em
questao, entao nao faz qualquer diferenca. Eu acho que o uso de intervalos
abertos na definicao de limite eh para garantir que o intervalo, ao conter
a, contenha pontos do dominio de f aa direita e aa esquerda de a, caso
existam.
Artur

 
Eu nao entendi esse argumento. De fato, acho que nao se usa um intervalo
fechado apenas porque um tal intervalo pode ser degenerado, ou seja,
consistir de um unico ponto (mais precisamente, um intervalo fechado pode
degenerar num conjunto unitario). No caso do limite de f(x) quando x - a, o
importante eh excluir o a da nossa analise, ou seja, estamos interessados
nos valores de f(x) com x proximo de a e diferente de a, e isso pode ser
feito tambem com um intervalko fechado (nao-degenerado).

Alem disso, todo intervalo fechado e nao-degenerado de centro em a e raio
epsilon contem um intervalo aberto centrado em a (de raio epsilon/2, por
exemplo).

[]s,
Claudio.


=
Instruções para entrar na lista, sair da lista e usar a lista em
http://www.mat.puc-rio.br/~nicolau/olimp/obm-l.html
=


OPEN Internet e Informática
@ Primeiro provedor do DF com anti-vírus no servidor de e-mails @


=
Instruções para entrar na lista, sair da lista e usar a lista em
http://www.mat.puc-rio.br/~nicolau/olimp/obm-l.html
=


Re: [obm-l] Re: [obm-l] dúvidas de limite e problema legal 6x6

2004-11-08 Por tôpico Artur Posenato
Desculpe-me por ter enviado posteriormente, minha
caixa de e-mail está bem devagar e ainda não tinha
recebido este seu e-mail.

Artur
--- Artur Costa Steiner [EMAIL PROTECTED] wrote:

 
 Oi amigos da lista! Gostaria de tirar umas dúvidas
 sobre Limites e mostrar
 uma questão legal.
 
 1) A definição de limite que eu vi foi feita em
 intervalo aberto. Por que em
 intervalo aberto? Poderia ser em intervalo fechado e
 se não por que?
 ex: Seja I um intervalo aberto ao qual pertence o
 número real a seja f uma
 função definida para x E I - {a}... (Gelson Iezzi,
 Fundamentos do Matemática
 Elementar).
 Eh importante que seja um intervalo aberto para
 garantir que a condicao
 |f(x) - L|  eps seja atendida nao importa como que
 x se aproxine de a. Se
 vc considerasse intervalos fechados, poderia nao ser
 possivel garantir esta
 condicao. Isto eh ainda mais visivel quando se tem
 funcoes definidas em r^n,
 n=2, pois x pode se aproximar de a segundo uma
 infinidade de possibilidaes.
 
 
 
 2) Uma dúvida na teoria do livro do iezzi. Numa
 parte ele fala sobre ser
 importante perceber que (delta) depende de
 (épsilon), não percebi isso e
 além de não perceber não vejo porque o (épsilon) não
 deva depender também do
 (delta)...
 Vc primeiro estavbelece arbitrariamente o valor de
 epsilon. Para este
 epsilon, vc tem que encontra um delta que satisfaca
 aa definicao de limite.
 De modo geral, o delta depende do epsilon o do valor
 de a no qual se avalia
 o limite. Isto eh, de modo geral, o valor de delta
 associadao a um epsilon
 que funciona para um dado a nao funciona para todos
 os pontos de acumulacao
 do dominio da funcao. Por exemplo, a funcao f(x) =
 x^2 apreenta limite em
 todo os elementos de R (eh continua), mas, fixado
 eps, a escolha do delta
 sempre vai depender de x. ja para a funcao
 identidade f(x) = x eh possivel,
 para um mesmo eps, achar um delta que funcione para
 todos o reais x. Isto
 esta ligado ao conceito de continuidade uniforme. 
 
 
 3) A demonstração do teorema da unicidade do limite,
 não entendi aquela do
 livro do iezzi por redução ao absurdo...
 (observação: sei o que é redução ao
 absurdo mais não entendi uma parte do
 desenvolvimento).
 
 Ele provavelmente fez algo deste tipo: Suponhamos
 que, em um ponto a, f
 apresente limites distintos L1 e L2. Seja r = |L1
 -L2|/2. Entao, r0 e os 
 intervalos abertos I1 e I2, de raio r e centros em
 L1 e L2, nao se
 intesectam. Pela definicao de limite, existem reais
 positivos d1 e d2 tais
 que,  f(x) estah em I1 se x estiver no dominio de f
 e 0|x-a| d1, e  f(x
 estah em I2 se x estiver no dominio de f e 0|x-a|
 d2. Temos entao que d =
 minimo{d1, d2} eh positivo e que, se x estiver no
 dominio de f e 0|x-a| d,
 entao f(x) estah em I1 e f(x) estah em I2. Isto
 signfica que I1 e I2 contem
 em comum o elemento f(x), contrariamente aa
 conclusao anterior de que sao
 disjuntos. Logo, o limite de f em um ponto de
 acumulacao de seu dominio, se
 existir, eh unico.
 
 Artur
 
 
 
 OPEN Internet e Informática
 @ Primeiro provedor do DF com anti-vírus no servidor
 de e-mails @
 
 

=
 Instruções para entrar na lista, sair da lista e
 usar a lista em
 http://www.mat.puc-rio.br/~nicolau/olimp/obm-l.html

=
 




__ 
Do you Yahoo!? 
Check out the new Yahoo! Front Page. 
www.yahoo.com 
 

=
Instruções para entrar na lista, sair da lista e usar a lista em
http://www.mat.puc-rio.br/~nicolau/olimp/obm-l.html
=


[obm-l] Re: [obm-l] Re: [obm-l] dúvidas de limite e problema legal 6x6

2004-11-08 Por tôpico Artur Costa Steiner
Desculpe-me por ter enviado posteriormente, minha
caixa de e-mail está bem devagar e ainda não tinha recebido este seu
e-mail.

Artur


Nao existe a mais leve razao para pedir desculpas!
O outro Artur


OPEN Internet e Informática
@ Primeiro provedor do DF com anti-vírus no servidor de e-mails @


=
Instruções para entrar na lista, sair da lista e usar a lista em
http://www.mat.puc-rio.br/~nicolau/olimp/obm-l.html
=


Re: [obm-l] Re: [obm-l] dúvidas de limite e problema legal 6x6

2004-11-08 Por tôpico Claudio Buffara
on 08.11.04 09:58, Artur Costa Steiner at [EMAIL PROTECTED] wrote:

 
 Oi amigos da lista! Gostaria de tirar umas dúvidas sobre Limites e mostrar
 uma questão legal.
 
 1) A definição de limite que eu vi foi feita em intervalo aberto. Por que em
 intervalo aberto? Poderia ser em intervalo fechado e se não por que?
 ex: Seja I um intervalo aberto ao qual pertence o número real a seja f uma
 função definida para x E I - {a}... (Gelson Iezzi, Fundamentos do Matemática
 Elementar).
 Eh importante que seja um intervalo aberto para garantir que a condicao
 |f(x) - L|  eps seja atendida nao importa como que x se aproxine de a. Se
 vc considerasse intervalos fechados, poderia nao ser possivel garantir esta
 condicao. Isto eh ainda mais visivel quando se tem funcoes definidas em r^n,
 n=2, pois x pode se aproximar de a segundo uma infinidade de possibilidaes.
 
 
Eu nao entendi esse argumento. De fato, acho que nao se usa um intervalo
fechado apenas porque um tal intervalo pode ser degenerado, ou seja,
consistir de um unico ponto (mais precisamente, um intervalo fechado pode
degenerar num conjunto unitario). No caso do limite de f(x) quando x - a, o
importante eh excluir o a da nossa analise, ou seja, estamos interessados
nos valores de f(x) com x proximo de a e diferente de a, e isso pode ser
feito tambem com um intervalko fechado (nao-degenerado).

Alem disso, todo intervalo fechado e nao-degenerado de centro em a e raio
epsilon contem um intervalo aberto centrado em a (de raio epsilon/2, por
exemplo).

[]s,
Claudio.


=
Instruções para entrar na lista, sair da lista e usar a lista em
http://www.mat.puc-rio.br/~nicolau/olimp/obm-l.html
=


[obm-l] Re: [obm-l] dúvidas

2004-04-25 Por tôpico TSD





  - Original Message - 
  From: 
  TSD 
  To: [EMAIL PROTECTED] 
  Sent: Sunday, April 25, 2004 2:54 
PM
  Subject: [obm-l] dúvidas
  
  POderiam explicar passo a passo cada questão Por 
  favor!!!
  1) A soma da sériee :
  1/2 + 1/3 + 1/4 + 1/9 + ...+1/(2^n) + 1/(3^n) + 
  1/(2^n+1) + 1/(3^n+1)+... =
  Somatório n=variando 1 ao infinito ( 1/(2^n) + 
  1/(3^n) )
  a)5/3 b)1 c)3/2 d)2 e)infinito
  
  está questão ´é do livro matemática elementar. 
  td(132)
  2) A soma dos termos da progressão 3^ -1 , 3 ^ -2 
  , 3^ -3,...e:
  a)1/2 b)2 c)1/4 d)4
  
  
  3) a expressão 1+ 2/2 + 3/4 + 4/8 + 5/16 + 
  ... vale ?
  a)1/2 b)9/2 c)1/4 
d)4


Re: [obm-l] Re: [obm-l] dúvidas

2004-04-25 Por tôpico Faelccmm
Para a 1)

S = 1/2 + 1/3 + 1/4 + 1/9 + ...+1/(2^n) + 1/(3^n) + 1/(2^n+1) + 1/(3^n+1)+...
S = (1/2 + 1/4 + 1/8 ...) + (1/3 + 1/9 + 1/27 ...)
S = 1 + 1/2 = 3/2


Para a 2)

S = 3^ -1 + 3 ^ -2 + 3^ -3 + ...3^-n
S = 1/3 + 1/3^2 + 1/27 + ...+1/3^n

Como trata-se de uma P.G infinita:

S = a[1] / 1 - q
S = 1/3 / 1 - 1/3
S = 1/2

Para 3)

S = 1+ 2/2 + 3/4 + 4/8 + 5/16 + ...

Eh uma P.G infinita. Ela pode ser vista como:

1/1
1/2 + 1/2
1/4 + 1/4 + 1/4 
1/8 + 1/8 + 1/8 + 1/8

.
.
.
Vamos somar a 1o coluna, depois a 2o coluna, depois a 3o coluna e veremos uma P.G cuja soma eh igual a S

s[1] = (1/1) / (1 - 1/2) = 2
s[2] = (1/2) / (1 - 1/2) = 1
s[3] = (1/4) / (1 - 1/2) = 1/2

Pronto ! Ja temos uma P.G, cujos termos sao 2,1,1/2 e cuja soma eh
S = 2 / (1 - 1/2) = 4 



Em uma mensagem de 26/4/2004 00:49:48 Hora padrão leste da Am. Sul, [EMAIL PROTECTED] escreveu:



 
- Original Message - 
From: TSD 
To: [EMAIL PROTECTED] 
Sent: Sunday, April 25, 2004 2:54 PM
Subject: [obm-l] dúvidas


POderiam explicar passo a passo cada questão Por favor!!!
1) A soma da sériee :
1/2 + 1/3 + 1/4 + 1/9 + ...+1/(2^n) + 1/(3^n) + 1/(2^n+1) + 1/(3^n+1)+... =
Somatório n=variando 1 ao infinito ( 1/(2^n) + 1/(3^n) )
a)5/3 b)1 c)3/2 d)2 e)infinito
 
está questão ´é do livro matemática elementar. td(132)
2) A soma dos termos da progressão 3^ -1 , 3 ^ -2 , 3^ -3,...e:
a)1/2 b)2 c)1/4 d)4
 
 
3) a expressão 1+ 2/2 + 3/4 + 4/8 + 5/16 + ... vale ?
a)1/2 b)9/2 c)1/4 d)4






[obm-l] Re: [obm-l] DÚVIDAS DE DÉCADAS

2004-04-21 Por tôpico Gabriel Reina
Robério Alves:
 OLÁ PESSOAL, RESPONDAM - ME ESSA
PERGUNTA ?
 EM QUE DÉCADA NÓS ESTAMOS  ATUALMENTE
?

Alan Pellejero:
 ESTAMOS DUZENTAS DÉCADAS APÓS O
NASCIMENTO DE CRISTO!
 rs..
 Boa Pergunta, mas se na década de
setenta foi nos anos
 setenta, então estaríamos na década 0,
pois ainda não
 chegamos ao dez...Talvez por isso,
evitamos falar que
 estamos na década zero...seria
estranho.

I. Anos setentas, não setenta.

II. Se a gente fala da década de
setenta, não parece
razoável que estejamos nos referindo à
década que contém
o ano de 1970? Dessa forma, a década de
setenta
corresponderia aos anos entre 1961 e
1970, e estaríamos
atualmente na década de dez.

Por outro lado, os anos setentas
poderiam corresponder a
todos os anos que começam com sete, e
daí teríamos o
intervalo entre 1970 e 1979 como os anos
setentas.

Não é algo para se pensar?

=
Instruções para entrar na lista, sair da lista e usar a lista em
http://www.mat.puc-rio.br/~nicolau/olimp/obm-l.html
=


[obm-l] Re: [obm-l] Re: [obm-l] DÚVIDAS DE DÉCADAS

2004-04-21 Por tôpico Gabriel Reina
From: Gabriel Reina
Subject: Re: [obm-l] DÚVIDAS DE DÉCADAS

Mas que raio de formatação foi essa?
Desculpem.

-- Gabriel

=
Instruções para entrar na lista, sair da lista e usar a lista em
http://www.mat.puc-rio.br/~nicolau/olimp/obm-l.html
=


[obm-l] Re: [obm-l] dúvidas - Correcao

2004-04-03 Por tôpico TSD



a 2º questão poderia explicar melhor por 
favor.

  - Original Message - 
  From: 
  [EMAIL PROTECTED] 
  
  To: [EMAIL PROTECTED] 
  Sent: Saturday, April 03, 2004 3:41 
  PM
  Subject: Re: [obm-l] dúvidas - 
  Correcao
  Nao sabia 
  que *genios* tbem tem historias etilicas ;-) brincadeirinha, super Claudio 
  Em uma mensagem de 3/4/2004 09:17:38 Hora padrão leste da Am. 
  Sul, [EMAIL PROTECTED] 
  escreveu: 
  
Troquei um sinal na minha solucao pro problema abaixo: 

  1)Um polinômio f do 3º grau e com coeficientes reais, é tal 
que f(-1)=0 e f(1+i)=0, sendo i a unidade imaginária. Se f(0)=--4. 
determine a soma dos coeficientes de f. 
  SOLUCAO CORRIGIDA: 1) As raizes de f 
  sao -1, 1 + i e 1 - i == f(x) = A*(x+1)*(x^2-2x+2) == 
  f(0) = A*(1)*(2) = -4 == A = -2 == Soma dos 
  coeficientes = f(1) = (-2)*(1+1)*(1-2+2) = -4. Nisso eh que 
  dah chegar bebado da noitada e ligar o computador... []s, 
  Claudio. 


[obm-l] Re: [obm-l] Re: [obm-l] dúvidas - Correcao

2004-04-03 Por tôpico Rafael
Vou tentar explicar de uma outra forma, ainda que algumas semelhanças sejam
inevitáveis.

(2^(1/3) + 3^(1/2))^10 =
= Sum[C(10,k) * 2^[(1/3)*(10-k)] * 3^[(1/2)*k], {k, 0, 10}] =
= Sum[C(10,k) * 2^[(10-k)/3] * 3^(k/2), {k, 0, 10}]

Não é muito difícil demonstrar, por redução ao absurdo, que a raiz enésima
de um número primo é irracional. Assim, como 2 e 3 são primos, precisamos
que os seus expoentes sejam inteiros. Temos que k varia de 0 a 10, então:

k/2 é inteiro == k pertence a {0,2,4,6,8,10}
(10-k)/3 é inteiro == k = 4 ou k = 10

Seja T_n o enésimo termo do desenvolvimento de (x+y)^n,

k = 4 == T_5 = C(10,4) * 2^[(10-4)/3] * 3^(4/2) = 7560
k = 10 == T_11 = C(10,10) * 2^[(10-10)/3] * 3^(10/2) = 243

Lembre-se de que o desenvolvimento de (x+y)^n tem n+1 termos. Assim, o
quinto e o décimo primeiro termos são os únicos cujo valor é racional.


Abraços,

Rafael de A. Sampaio




- Original Message -
From: TSD
To: [EMAIL PROTECTED]
Sent: Saturday, April 03, 2004 9:07 AM
Subject: [obm-l] Re: [obm-l] dúvidas - Correcao


a 2º questão poderia explicar melhor por favor.

=
Instruções para entrar na lista, sair da lista e usar a lista em
http://www.mat.puc-rio.br/~nicolau/olimp/obm-l.html
=


[obm-l] Re: [obm-l] dúvidas

2004-02-24 Por tôpico Nicolau C. Saldanha
On Tue, Feb 24, 2004 at 02:37:44AM -0300, Tarcio Santiago wrote:
 1) Quantos elementos tem o conjunto dos bisavós dos meus bisavós (bisavós são
 os pais de seus avós): A) 16 B)  32 C)  64 D)  81

O único comentário que eu tenho a fazer é que a resposta óbvia (64)
já tem boa probabilidade de estar errada com 6 gerações. Afinal,
ninguém consideraria incesto casar e ter filhos com alguém que tem
o mesmo avô-do-bisavô. Aliás, em geral nós não temos a menor idéia
de quem são as pessoas que tem algum avô-do-bisavô em comum conosco.

Eu marcaria 64, mas pode ser qualquer número menor do que isso e maior
do que um mínimo razoável e difícil de precisar; o mínimo absoluto é 2.

Note que ao aumentarmos o número de gerações (quantos elementos tem
o conjunto dos bisavós dos bisavós dos bisavós dos bisavós ...
dos meus bisavós?) chega um ponto em que a resposta 2^n fica claramente
errada pois fica maior do que o número de seres humanos que viveram
no período relevante da história.

[]s, N.
=
Instruções para entrar na lista, sair da lista e usar a lista em
http://www.mat.puc-rio.br/~nicolau/olimp/obm-l.html
=


[obm-l] Re: [obm-l] Dúvidas

2004-02-20 Por tôpico Rafael



Pedro,

Para o problema 1, sabemos que a equação de uma 
reta tangenteé dada por: y = mx + h.

De x^2 + y^2 = 25 e y = mx + h, chegamos a 
(m^2+1)x^2 + (2mk)x + k^2 - 25 = 0, que é quadrática em x. Impondo que o 
discriminante seja igual a zero, visto que há um único ponto de tangência. 
Obter-se-á a equação 25m^2 - k^2 + 25 = 0.

Mas sabemos que o ponto (5,-6) pertence à reta y = 
mx + h, então 5m + k = -6, ou ainda, k = - 5m - 6.

De 25m^2 - k^2 + 25 = 0 e k = - 5m - 6, chegamos a 
m = - 11/60 e k = - 61/12. Logo, a equação de uma das tangentes é dada por y = 
-11/60x - 61/12. 

Como o coeficiente angular encontrado é único, e 
para cada ponto há duas tangentesà circunferência, devemosconcluir 
que a outra reta não só é tangente à circunferência, mas também perpendicular ao 
eixo x. Assim, a outra tangente é dada pela equação x = 5.

De x^2 + y^2 = 25 e x = 5, obtemos o primeiro ponto 
de tangência: (5,0).
De x^2 + y^2 = 25 e y =- 11/60x - 61/12, 
obtemos o outro ponto de tangência: (-55/61, -300/61).

Calculando a distância D entre esses pontos: 
D= sqrt((5+55/61)^2+(0+300/61)^2) = 60*sqrt(61)/61.

Como o exercício pede a metade do comprimento da 
corda que une os pontos, em outras palavras, metade da distância determinada por 
eles: D/2 = 30*sqrt(61)/61. Alternativa B.


Para o problema 2, por hipótese, a equação da 
elipse é dada por x^2/16 + y^2/9 = 1. Na verdade, não faz diferença para o 
problema se optássemospela equação x^2/9 + y^2/16 = 1 
(translação).

Como desejamos obter o maior valor possível para x 
e y tal que o quadrado possa ser inscrito na elipse, devemos impor: x = y == x^2- y^2 = 0. 

De x^2/16 + y^2/9 = 1 e x^2 - y^2= 0, obtemos 
quatro soluções que são osvértices do quadrado. Assim, para o primeiro 
quadrante, temos o ponto (12/5,12/5). A medida do lado do quadrado é, pois, 24/5 
e, por conseguinte, a sua área é 576/25. Alternativa B.


Abraços,

Rafael de A. Sampaio




  - Original Message - 
  From: 
  Pedro Costa 
  To: [EMAIL PROTECTED] 
  Sent: Friday, February 20, 2004 10:04 
  AM
  Subject: [obm-l] Dúvidas
  
  1) Em um sistema de coordenadas 
  ortogonais no plano são dados o ponto (5,-6) e a circunferênciax^2 + y^2 = 25. A partir do ponto 
  (5,-6), traçam-se duas tangentes a circunferência. Calcule a metade do 
  comprimento da corda que une os pontos de tangência. 
  a) b) c) d) 
  2) Dada uma elipse, com centro na origem, de semi-eixo a=4 e b=3, calcule a 
  área do quadrado nela inscrito, com lados paralelos aos eixos da elipse.
  a) b) 
  c) d) 
  
clip_image002.gifclip_image004.gifclip_image006.gifclip_image008.gifclip_image010.gifclip_image012.gifclip_image014.gifclip_image016.gif

[obm-l] Re: [obm-l] Dúvidas !!!

2004-01-19 Por tôpico dasilvalg
Ricardo respondeu:

 dasilvalg wrote:
 
  4) Ache os dois últimos algarismos de 2^1997.
  Obs.: Neste exercício só consegui achar o último 
  algarismo (unidades) que é 2, mas o das dezenas não te
nho 
  nem idéia.
 
   Quem trabalha com computação sabe de cabeça
 que 1 megabyte = 1048576 bytes. Logo 2 mega = 2097152
 e 4 mega = 4194304. Como 4 mega = 2^22, temos que
 2^22 = 2^2 (mod 100) e portanto 2^(20k+n)=2^n (mod 100)
 para k=1.
 
   Agora fica fácil, 2^1997=2^17 (mod 100),
 e chegamos em 2^17=131072=72 (mod 100), concluíndo
 que 2^1997 termina em 72.
 
 

 Ricardo Bittencourt   http://www.mundobi
zarro.tk
 [EMAIL PROTECTED]   tenki ga ii kara sanpo sh
imashou
 -- União contra o forward -
 crie suas proprias piadas --
 
 
=
 Instruções para entrar na lista, sair da lista e usar a 
lista em
 http://www.mat.puc-rio.br/~nicolau/olimp/obm-l.html
 
=


Nao entendi esta congruencia. Por que que se 
2^22 = 2^2 (mod 100) = 2^(20k + n) = 2^n (mod 100) para 
k = 1. Foi mal, literalmente, BOIEI !!!

Abracos!!!

 

 
__
Acabe com aquelas janelinhas que pulam na sua tela.
AntiPop-up UOL - É grátis!
http://antipopup.uol.com.br/



=
Instruções para entrar na lista, sair da lista e usar a lista em
http://www.mat.puc-rio.br/~nicolau/olimp/obm-l.html
=


Re: [obm-l] Re: [obm-l] Dúvidas !!!

2004-01-19 Por tôpico Ricardo Bittencourt
dasilvalg wrote:
Nao entendi esta congruencia. Por que que se 
2^22 = 2^2 (mod 100) = 2^(20k + n) = 2^n (mod 100) para 
k = 1. Foi mal, literalmente, BOIEI !!!
	É que você pode multiplicar por 2 nos dois lados:

2^22 = 2^2 (mod 100)
2* 2^22 = 2* 2^2 (mod 100)
2^23 = 2^3 (mod 100)
Agora repetindo o processo:
2^23 = 2^3 (mod 100)
2^24 = 2^4 (mod 100)
2^25 = 2^5 (mod 100)
2^26 = 2^6 (mod 100)
2^27 = 2^7 (mod 100)
...
2^42 = 2^22 (mod 100)
mas 2^22 = 2^2 então aqui fica periódico
2^42 = 2^22 = 2^2 (mod 100)
e se o período é 20, então fica claro que
2^(20k + n) = 2^n (mod 100)
deixando claro que isso só vale a partir da primeira
vez que repete, que é pra k=1. Pra k=0 dá errado, por exemplo
2^1=2 (mod 100) e 2^21=52 (mod 100)

Ricardo Bittencourt   http://www.mundobizarro.tk
[EMAIL PROTECTED]   tenki ga ii kara sanpo shimashou
-- União contra o forward - crie suas proprias piadas --
=
Instruções para entrar na lista, sair da lista e usar a lista em
http://www.mat.puc-rio.br/~nicolau/olimp/obm-l.html
=


[obm-l] RE: [obm-l] Dúvidas !!!

2004-01-18 Por tôpico Paulo Santa Rita
Ola dasilvag e demais
colegas desta lista ... OBM-L,
Bem-Vindo a Lista OBM-L ! Evite usar sinais ortograficos, caracteres 
especiais e/ou textos com
formatacoes de algum software especifico, pois os podem enunciados ficam 
ilegiveis com eles.

Um Abraco
Paulo Santa Rita
1,1041,180104
From: dasilvalg [EMAIL PROTECTED]
Reply-To: [EMAIL PROTECTED]
To: [EMAIL PROTECTED]
Subject: [obm-l] Dúvidas !!!
Date: Sat, 17 Jan 2004 19:42:19 -0200
Me chamo Leonaro e é a primeira vez que mando uma
mensagem para a lista. Se for possível uma ajuda,
agradeço desde já.
Um abraço e até a próxima.
Segue aí uns 10 probleminhas:

1) Seja uma função F:Z+*#8594;Z+, atendendo às seguintes
condições:
a) F(m*n) = F(m) + F(n);
b) F(n) = 0, se o último algarismo de n é 3;
c) F(10) = 0.
Demonstre que F(n) = 0 para todo inteiro positivo n.

2) Verifique que n^2 + 3n + 5 nunca é divisível por 121,
qualquer que seja n.
3) Seja D = {(x, y) #1028; R2 | 0  x 1, 0  y  1} e F:D#8594;R2
uma função tal que V(significa: para todo) (x, y) #1028; D
associa (X, Y) #1028; R2 onde X = y e Y = (1 – y)x.
a) Sendo T = {(X, Y) | X  0, Y   0, X + Y  1}, mostre
que F é uma bijeção de D sobre T;
b) Esboce a imagem dos conjuntos da forma {(x, y) #1028; D |
y = #955;x} para os seguintes valores de #955;: #955; = ¼, #955;' = ½,
#955;'' = 1.
4) Ache os dois últimos algarismos de 2^1997.
Obs.: Neste exercício só consegui achar o último
algarismo (unidades) que é 2, mas o das dezenas não tenho
nem idéia.
5) Seja F:N#8594;N tal que F(1) = 1, F(2k +1) = F(2k) + 1, F
(2k) = 2F(k), k #1028; N. Determine F(n) em função de n.
6) rc(x + rc(1- x)) = 3/2.
Obs.: rc quer dizer raiz cúbica.
7) Prove que existem 2[2^(n-1) – 1] maneiras distintas de
se distribuir n cartas para dois jogadores.
Obs.: Os jogadores devem receber o mesmo número de cartas.
 8) Se um quadrilátero cujos lados medem a, b, c e x está
inscrito num semi-círculo de diâmetro x, então:
x^3 – (a^2 + b^2 +c^2)x – 2abc = 0

9) Em um país, as distâncias entre todas as suas cidades
são distintas duas a duas. Certo dia, de cada cidade
parte um avião, com destino à cidade mais próxima.
Demonstre que em nenhuma cidade aterrissaram mais de 5
aviões.
10) Prove que log n  k*log 2 , onde n é um número
natural e k é o número de primos distintos que dividem n.
Obs.: Log é a função logarítimica na base 10.
_
MSN Messenger: converse com os seus amigos online.  
http://messenger.msn.com.br

=
Instruções para entrar na lista, sair da lista e usar a lista em
http://www.mat.puc-rio.br/~nicolau/olimp/obm-l.html
=


[obm-l] Re: [obm-l] Re: [obm-l] Dúvidas

2004-01-16 Por tôpico Nicolau C. Saldanha
On Thu, Jan 15, 2004 at 11:31:57PM -0200, Rafael wrote:
 Infelizmente, alguns autores não consideram o zero como sendo
 um número positivo ou negativo.

Na França, 0 é considerado positivo *e* negativo.
Assim, para Bourbaki o conjunto dos inteiros positivos é {0,1,2,...}
Se você quer excluir o zero, você deve dizer 'estritamente positivo'
ou 'estritamente negativo'.

Para o resto do mundo, 0 não é nem positivo nem negativo.
Assim, para a maioria do mundo o conjunto dos inteiros positivos
é {1,2,3,...}. O conjunto {0,1,2,3,...} pode ser chamado de
conjunto dos inteiros não negativos.

 Ou, mais rigorosamente, não o consideram como número algébrico.

Isto eu acho bem mais estranho.

Para mim um número algébrico é um número que é raiz de algum polinômio não nulo
com coeficientes inteiros. Assim, por exemplo, sqrt(2) + sqrt(3) é algébrico
pois é raiz do polinômio x^4 - 10 x^2 + 1 mas o número pi não é algébrico
pois não existe nenhum polinômio não nulo de coeficientes inteiros com a raiz
pi (mas isto não é tão fácil de demonstrar). Neste sentido é bem óbvio que
o número 0 é algébrico: é raiz do polinômio x.

Existem definições equivalentes: por exemplo, podemos dizer que x é algébrico
se existe um corpo K com x em K, Q um subcorpo de K e K de dimensão finita
se considerado como Q-espaço vetorial. Mas nunca vi a expressão 'número
algébrico' ser usada para um conceito realmente diferente e eu acharia
péssima idéia fazer isso.

[]s, N.
=
Instruções para entrar na lista, sair da lista e usar a lista em
http://www.mat.puc-rio.br/~nicolau/olimp/obm-l.html
=


[obm-l] Re: [obm-l] Dúvidas !!!

2004-01-16 Por tôpico dasilvalg
Foi mal, eu copiei errado, faltava o sinal de =. Valeu.

Ricardo respondeu:
 dasilvalg wrote:
  10) Prove que log n  k*log 2 , onde n é um número 
  natural e k é o número de primos distintos que dividem
 n.
  Obs.: Log é a função logarítimica na base 10. 
 
   Mas isso aqui não é verdade, né?
 
   Pegue por exemplo n=2, nesse caso k=1 e teríamos
 log 2  1.log 2 = log 2  log 2, que é falso. Se você
 consertar o enunciado pra log n = k*log 2, aí sim o pro
blema
 tem solução.
 
 

 Ricardo Bittencourt   http://www.mundobi
zarro.tk
 [EMAIL PROTECTED]   tenki ga ii kara sanpo sh
imashou
 -- União contra o forward -
 crie suas proprias piadas --
 
 
=
 Instruções para entrar na lista, sair da lista e usar a 
lista em
 http://www.mat.puc-rio.br/~nicolau/olimp/obm-l.html
 
=
 

 
__
Acabe com aquelas janelinhas que pulam na sua tela.
AntiPop-up UOL - É grátis!
http://antipopup.uol.com.br/



=
Instruções para entrar na lista, sair da lista e usar a lista em
http://www.mat.puc-rio.br/~nicolau/olimp/obm-l.html
=


Re: [obm-l] Re: [obm-l] Dúvidas !!!

2004-01-16 Por tôpico Ricardo Bittencourt
dasilvalg wrote:

Foi mal, eu copiei errado, faltava o sinal de =. Valeu.
	Ah, então agora dá pra continuar. Fatore n como:

	n=p1^a1 . p2^a2 .  pn^an

	Sabemos que pj^aj = pj = 2, logo

n=p1^a1 . p2^a2 .  pn^an
n= p1 . p2 . ... pn
n= 2 . 2 . . 2
n= 2^k
log n = k log 2 QED

Ricardo Bittencourt   http://www.mundobizarro.tk
[EMAIL PROTECTED]   tenki ga ii kara sanpo shimashou
-- União contra o forward - crie suas proprias piadas --
=
Instruções para entrar na lista, sair da lista e usar a lista em
http://www.mat.puc-rio.br/~nicolau/olimp/obm-l.html
=


[obm-l] Re: [obm-l] Dúvidas

2004-01-15 Por tôpico Rafael



Pedro,

Infelizmente, alguns autores não consideram o zero 
como sendoum número positivo ou negativo. Ou, mais rigorosamente, não o 
consideram como número algébrico. Como você pediu uma explicação 
detalhada,veja:

Para a + b + c = 7,vêm:

0 + 0 + 7 = 7é 
solução,portanto aspermutações dessestrês números (dois 
repetidos) representam:3!/2! = 3 soluções0 + 1 + 6 = 7é 
solução,não havendo números repetidos,temos as permutações de três 
números, isto é,3! = 6 soluções0 + 2 + 5 = 7é solução, de 
forma análoga,3! = 6 soluções0 + 3 + 4 = 7ésolução, 
assim, temos: 3! = 6 soluções

Observe que o número total de soluções em que a ou b ou 
c é zero será: 3 + 6 + 6 + 6 = 21
De maneira semelhante, ainda teremos:


1 + 1 + 5 = 7 3!/2! = 3 soluções1 + 2 + 4 = 
7 3! = 6 soluções1 + 3 + 3 = 
7 3!/2! = 3 soluções2 + 2 + 3 = 
7 3!/2! = 3 soluções

Agora,a e b e c são diferentes de zero. O número 
total de soluções será: 3 + 6 + 3 + 3 = 15

Não é difícil concluir o que disse eu inicialmente: quando o autor se 
referiu a "soluções inteiras positivas", ele tinha em mente que as três 
incógnitas fossem diferentes de zero; ao dizer "soluções inteiras não 
negativas", ele intencionava que uma das incógnitas, pelo menos,fosse 
igual a zero. Logo, o item (a) corresponde a 15 soluções e o item (b) 
corresponde a 21+15=36 soluções.
E, de fato, não é fácil saber o que o "autor tem em mente" ao elaborar uma 
questão. Como se diz, quem quer uma boa resposta deve fazer uma boa pergunta. 
Como a questão consta de dois itens, quando lemos, temos a impressão de que o 
autor está louco perguntando duas vezes a mesma coisa (o que não costuma 
ocorrer...), ou que há algum detalhe que estamos perdendo...

Uma outra dúvida freqüente está no conjunto dos números naturais:o 
zero pertence ou não a ele? A maioriadiz que sim, outros, entretanto, 
não.O zero, ou ausência de objetos num conjunto, não era em geral aceito 
como número antes do século XIII.A explicação, certa vez li, estáem 
quealguns constroem os naturais por axiomas de Peanno, enquanto outros, 
não.


Enfim, espero ter podido ajudá-lo e, qualquer dúvida, não hesite: pergunte! 
;-)


Um forte abraço,

Rafael de A. Sampaio



  - Original Message - 
  From: 
  Pedro Costa 
  To: [EMAIL PROTECTED] 
  Sent: Sunday, January 11, 2004 7:58 
  PM
  Subject: [obm-l] Dúvidas
  
  Estudando análise combinatória , tive uma dúvida 
  , como vocês são geniais me ajudem.
  
   Dada a equação a + b + c = 7 , 
  calcule:
  
   a) O número de soluções inteiras 
  positivas. R.15
   b) O número de soluções inteiras não 
  negativa. R.36
  
  
  As dúvidas:
  
  a) soluções 
  inteiras positivas = soluções inteiras não negativa , Essas afirmações 
  não é as mesma?
   No livro: 
  prelúdio à análise combinatória essas afirmações são 
  diferente, porque? qual o propósito? não 
  são apenas sinônimos ? Se possível uma 
  explicação detalhada.-- Esta mensagem foi verificada pelo 
  sistema de antivírus e acredita-se estar livre de perigo. 



[obm-l] Re: [obm-l] Dúvidas

2004-01-11 Por tôpico Bruno Souza



Pedro,
Quando se diz em números não negativos estamos se 
referindo aos positivos e ao ZERO.
Quando se diz em números positivos estamos se 
referindo aos números estritamentes positivos, lembrando que o ZERO não é 
positivo nem negativo, e sim neutro
Acho q essa era dúvida, neh?
Até,
Bruno...

  - Original Message - 
  From: 
  Pedro Costa 
  To: [EMAIL PROTECTED] 
  Sent: Sunday, January 11, 2004 7:58 
  PM
  Subject: [obm-l] Dúvidas
  
  Estudando análise combinatória , tive uma dúvida 
  , como vocês são geniais me ajudem.
  
   Dada a equação a + b + c = 7 , 
  calcule:
  
   a) O número de soluções inteiras 
  positivas. R.15
   b) O número de soluções inteiras não 
  negativa. R.36
  
  
  As dúvidas:
  
  a) soluções 
  inteiras positivas = soluções inteiras não negativa , Essas afirmações 
  não é as mesma?
   No livro: 
  prelúdio à análise combinatória essas afirmações são 
  diferente, porque? qual o propósito? não 
  são apenas sinônimos ? Se possível uma 
  explicação detalhada.-- Esta mensagem foi verificada pelo 
  sistema de antivírus e acredita-se estar livre de perigo. 



[obm-l] Re: [obm-l] dúvidas

2003-12-28 Por tôpico Henrique Patrício Sant'Anna Branco
 5)

 5x= 2*pi - x
 6x = 2*pi
 x=pi/3 = 60º

Tentei expandir cos(5x) e resultou em
16cos(x)^5 - 20cos(x)^3 + 5cos(x) = cos(x) ==
4cos(x)^5 - 5cos(x)^3 + cos(x) = 0

Fazendo a = cos(x), temos
4a^5 - 5a^3 + a = 0
Claramente, a = 0 é uma solução (e, portanto, x = Pi/2), segue
4a^4 - 5a^2 + 1 = 0
É fácil ver também que a = 1 é uma solução (a soma dos coeficientes é igual
a zero) e x = 0.
Mas ao invés de dividir por (a - 1), vamos resolver a equação biquadrada,
fazendo b = a^2

Temos então
4b^2 - 5b + 1 = 0.
Pela nossa conhecida fórmula pra equações do segundo grau, achamos b = 1 e b
= 1/4. Voltando as variáveis, achamos a = 1, a = -1, a = 1/2 e a = -1/2.
Pra a = -1, x = Pi; a = 1/2, x = Pi/3 e, finalmente, a = -1/2, x = 2Pi/3.

Acho que tá certo. Deve ter um jeito mais bonito de fazer isso, mas não
encontrei.

Abraços,
Henrique.

=
Instruções para entrar na lista, sair da lista e usar a lista em
http://www.mat.puc-rio.br/~nicolau/olimp/obm-l.html
=


[obm-l] RE: [obm-l] dúvidas

2003-12-28 Por tôpico Douglas Bokliang
5)

cos5x=cosx
cos5x-cosx=0
-2*sen3x*sen2x=0
sen3x=0 sen2x=0
3x=k*pi  sen2x=k*pi
x=k*pi/3 x=k*pi/2
substituindo os valore de k: S={0;pi/3,pi/2;pi;2pi/3;4pi/3;3pi/2;5pi/3}

espero q esteja certu

abracos
Douglas

From: [EMAIL PROTECTED]
Reply-To: [EMAIL PROTECTED]
To: [EMAIL PROTECTED]
Subject: [obm-l] dúvidas
Date: Sun, 28 Dec 2003 13:21:41 -0200
1)UM cone reto tem altura de 12cm e esta cheio de sorvete. Dois amigos vão
dividir o sorvete em duas partes de mesmo volume, usando um plano paralelo 
a
base do cone. \Qual deverá ser a altura do cone menor assim obtido?

2) Num recipiente cônico, completamente cheio de água, foi introduzida uma
esfera maciça de raio 3. Determine o volume de água que permaneceu no
recipiente. dado a altura do cone é 8.
3) num lançamento de dois dados diferentes, qual a probabilidade de que
saiam números diferentes?
4)o número de maneiras com se pode separar as letras da palavra PERNAMBUCO
em 2 conjuntos de 5 letras cada, de forma que as letras P e O não estejam 
no
mesmo conjunto.

5)Determine o conjunto solução da equação
 cos5x = cosx em [0,2pi[
_
Voce quer um iGMail protegido contra vírus e spams?
Clique aqui: http://www.igmailseguro.ig.com.br
Ofertas imperdíveis! Link: http://www.americanas.com.br/ig/
=
Instruções para entrar na lista, sair da lista e usar a lista em
http://www.mat.puc-rio.br/~nicolau/olimp/obm-l.html
=
_
MSN Messenger: converse com os seus amigos online.  
http://messenger.msn.com.br

=
Instruções para entrar na lista, sair da lista e usar a lista em
http://www.mat.puc-rio.br/~nicolau/olimp/obm-l.html
=


[obm-l] RE: [obm-l] DÚVIDAS/HOMOMORFISMO

2003-12-03 Por tôpico Paulo Santa Rita
Oi Douglas e demais
colegas desta lista ... OBM-L,
Homomorfismo de um grupo em si mesmo tem nome proprio ...

f e isomorfismo :
f(ab)=f(a)f(b)=(a^-1)(b^-1)=(ba)^-1=f(ba)  = ab=ab  = abeliano
G e abeliano
f(ab)=(ab)^-1=(b^-1)(a^-1) = f(b)f(a) = f(a)f(b)  = isomorfismo
Um outro de Grupo :

Se G e um grupo e G' e o grupo dos comutadores, G/G´ e abeliano.

Um Abraco
Paulo Santa Rita
4,1721,031203

From: [EMAIL PROTECTED]
Reply-To: [EMAIL PROTECTED]
To: [EMAIL PROTECTED]
Subject: [obm-l] DÚVIDAS/HOMOMORFISMO
Date: Wed, 3 Dec 2003 15:43:40 -0300
MIME-Version: 1.0
Received: from mc1-f38.hotmail.com ([64.4.50.45]) by mc1-s19.hotmail.com 
with Microsoft SMTPSVC(5.0.2195.6713); Wed, 3 Dec 2003 10:55:03 -0800
Received: from saci.mat.puc-rio.br ([139.82.27.51]) by mc1-f38.hotmail.com 
with Microsoft SMTPSVC(5.0.2195.6713); Wed, 3 Dec 2003 10:54:17 -0800
Received: from saci.mat.puc-rio.br (localhost [127.0.0.1])by 
saci.mat.puc-rio.br (8.12.8/8.12.8) with ESMTP id hB3IiPSU016400for 
[EMAIL PROTECTED]; Wed, 3 Dec 2003 16:44:25 -0200
Received: (from [EMAIL PROTECTED])by saci.mat.puc-rio.br 
(8.12.8/8.12.8/Submit) id hB3IiPIk016398for obm-l-MTTP; Wed, 3 Dec 2003 
16:44:25 -0200
Received: from summer6.zipmail.com.br (smtp.zipmail.com.br 
[200.221.11.147])by saci.mat.puc-rio.br (8.12.8/8.12.8) with ESMTP id 
hB3IiOSU016394for [EMAIL PROTECTED]; Wed, 3 Dec 2003 16:44:24 -0200
Received: from [143.107.230.1] by summer6.zipmail.com.br with HTTP; Wed, 3 
Dec 2003 16:43:40 -0200
X-Message-Info: SuyIeF3cBu9Pz1okjMdDJeXL063+Fx0yP6xSkvxaVXs=
Message-ID: [EMAIL PROTECTED]
X-MIME-Autoconverted: from quoted-printable to 8bit by saci.mat.puc-rio.br 
id hB3IiOSU016396
Precedence: bulk
Return-Path: [EMAIL PROTECTED]
X-OriginalArrivalTime: 03 Dec 2003 18:54:18.0750 (UTC) 
FILETIME=[DB2459E0:01C3B9CE]

Alguém poderia me esclarecer uma dúvida sobre homomorfismo?

Seja G um grupo. Mostre que a aplicação f : G - G, definida por f(x)=x^-1
(qualquer x pertencente a G), é um homomorfismo se e somente se G é 
abeliano.

Grato.

Douglas



--
Use o melhor sistema de busca da Internet
Radar UOL - http://www.radaruol.com.br


=
Instruções para entrar na lista, sair da lista e usar a lista em
http://www.mat.puc-rio.br/~nicolau/olimp/obm-l.html
=
_
MSN Messenger: converse com os seus amigos online.  
http://messenger.msn.com.br

=
Instruções para entrar na lista, sair da lista e usar a lista em
http://www.mat.puc-rio.br/~nicolau/olimp/obm-l.html
=


[obm-l] Re: [obm-l] dúvidas

2003-11-16 Por tôpico Igor GomeZZ

Em 16/11/2003, 15:23, tarciosd ([EMAIL PROTECTED]) disse:

 olá amigos quanto vale a soma dos 100 primeiros termos de: 
 os problemas estavam faltando um parenteses. 

 a)cosa,cos(a+pi),cos(a+2pi)..

cosa + cos(a+pi) + cos(a+2pi) + ... + cos(a + 99pi) = S

cosa - cosa + cosa - cosa + ... - cosa = S

01  2  399

Positivo ser for um termo par e negativo se o contrário, logo, existem 50
postivos e 50 negativos, a soma vale zero

S = 0

 b) cosa,cos²a,cos³a,...

cosa + cosa^2 + cosa^3 + ... + cosa^100 = S

cosa(1 + cosa + cosa^2 + cosa^3 + ... + cosa^99) = S

cosa(1 + S - cosa^100) = S

cosa + Scosa - cosa^101 = S

S = (cosa - cosa^101)/(1 - cosa)


 c)cosa,cos(a+pi)²,cos(a+2pi)³... 

cosa + cos(a+pi)^2 + cos(a+2pi)^3 + ... + cos(a+99pi)^100 = S

cosa + (-cosa)^2 + (cosa)^3 + (-cosa)^4 + ... + (-cosa)^100 = S

cosa + cosa^2 + cosa^3 + cosa^4 + ... + cosa^100 = S

Que eh igual à anterior:

S = (cosa - cosa^101)/(1 - cosa)

Ateh


   Igor GomeZZ   
 ICQ#: 29249895
 Vitória, Espírito Santo, Brasil
 Criação: 16/11/2003 (16:35)
#
Pare para pensar:

Seja igual e junte-se a sociedade, seja diferente e 
torne-se um líder. (Autor desconhecido)

#


=
Instruções para entrar na lista, sair da lista e usar a lista em
http://www.mat.puc-rio.br/~nicolau/olimp/obm-l.html
=


[obm-l] Re: [obm-l] Dúvidas

2003-09-22 Por tôpico Cláudio \(Prática\)



Dois probleminhas:

1) Qual é a soma dos algarismos do produto em que 
os fatores são um número constituído por 45 algarismos iguais a 9 e o outro, um 
número cosntituído por 45 algarismos iguais a 5?

Considere o polinômio:
P(x) = [9*(x^44 + x^43 + ... + x + 1)]*[5*(x^44 + 
x^43 + ... + x + 1)] =
45*(x^44 + x^43 + ... + x + 1)^2 

O valor do produto pedido é igual a P(10) e a soma 
dos seus algarismos é P(1).

Logo, Soma = P(1) = 45*(1 + 1 + ... + 1 + 1)^2 = 
45*45^2 = 45^3 = 91125.

*

2)Há 50 bolas brancas e 50 bolas pretas que 
serãodistribuídasem duas urnas idênticas. Em seguida, um prisioneiro 
escolherá (aleatoriamente) uma dessas urnas e dela retirará uma bola. Se esta 
for preta,o prisioneiroé condenado à morte. Caso contrário,o 
prisioneiroé libertado. Que distribuição de bolas pelas urnas dará ao 
prisioneiro a menor probabilidade de ele ser condenado à morte?

A distribuição ótima é: 
Urna 1: 1 bola branca e nenhuma preta.
Urna 2: 49 bolas brancas e 50 bolas 
pretas.


Um abraço,
Claudio.


[obm-l] Re: [obm-l] Dúvidas básicas...

2003-02-16 Por tôpico Wagner
Oi para todos !

x^0 indica a identidade do grupo C* para a operação multiplicação.
Logo a.x^0 = a , para todo a pertencente a C* . Logo x^0 = 1

André T.



- Original Message -
From: Henrique Branco [EMAIL PROTECTED]
To: [EMAIL PROTECTED]
Sent: Thursday, February 13, 2003 8:42 PM
Subject: [obm-l] Dúvidas básicas...


 Pessoal,
 Tenho duas dúvidas que são bem básicas...
 Existe alguma demonstração (formal, de preferencia) sobre x^0 = 1 e 0! =
1?
 Sendo 0! o fatorial de zero.
 Grato,
 Henrique.

 =
 Instruções para entrar na lista, sair da lista e usar a lista em
 http://www.mat.puc-rio.br/~nicolau/olimp/obm-l.html
 O administrador desta lista é [EMAIL PROTECTED]
 =


=
Instruções para entrar na lista, sair da lista e usar a lista em
http://www.mat.puc-rio.br/~nicolau/olimp/obm-l.html
O administrador desta lista é [EMAIL PROTECTED]
=



[obm-l] Re: [obm-l] Dúvidas básicas...

2003-02-14 Por tôpico Nicolau C. Saldanha
On Thu, Feb 13, 2003 at 08:42:40PM -0300, Henrique Branco wrote:
 Pessoal,
 Tenho duas dúvidas que são bem básicas...
 Existe alguma demonstração (formal, de preferencia) sobre x^0 = 1 e 0! = 1?

Isto são definições, não é possível propriamente demonstrá-las.
O que se pode fazer é mostrar pq estas são as definições apropriadas,
as únicas que levam certas propriedades a serem satisfeitas.

Por exemplo, se você desejar que a propriedade x^(a+b) = x^a x^b
seja sempre satisfeita, você deve ter x^0 = x^0 x^0 donde x^0 = 0 ou 1.
Mas se x^0 = 0 temos x^a = x^0 x^a = 0 para todo a,
o que não é muito interessante...

Analogamente, você provavelmente gosta da fórmula (n+1)! = n! (n+1).
Desta fórmula sai que 1! = 0! * 1; 2! = 0! * 1 * 2; e a única forma
de obtermos os valores esperados 2!=2, 3!=6 é definir 0!=1.

Outra idéia é levar interpretações combinatórias até casos degenerados.
Por exemplo, se a e b são inteiros positivos podemos escrever a^b = |F(B,A)|,
onde A e B são conjuntos arbitrários com a e b elementos, respectivamente,
e F(B,A) é o conjunto das funções de domínio B e contradomínio A.
Ora, se B for vazio, o que é F(B,A)? É um conjunto com um único elemento,
a função vazia. Eu vou dar os valores dela em todos os elementos de B;
pronto, já dei. :-) Assim, |F(B,A)| = 1 o que nos leva a x^0 = 1.

Analogamente, podemos escrever a! como |S(A)| onde A novamente é um conjunto
com A elementos e S(A) é o conjunto das permutações de A (bijeções de A em A).
Se A for vazio, novamente a função vazia é o único elemento de S(A).

[]s, N.



=
Instruções para entrar na lista, sair da lista e usar a lista em
http://www.mat.puc-rio.br/~nicolau/olimp/obm-l.html
O administrador desta lista é [EMAIL PROTECTED]
=



[obm-l] Re: [obm-l] Dúvidas básicas...

2003-02-13 Por tôpico David Ricardo

1) (x^1 / x^1) = x^(1-1) = x^0 = 1.
2) Quanto ao 0! eu não sei... Mas posso tentar uma resposta (nao sei se é
realmente válida): 1! = 1 = 1! = 1 * 0! = 1 = 0! = 1

[]s
David

___
Busca Yahoo!
O serviço de busca mais completo da Internet. O que você pensar o Yahoo! encontra.
http://br.busca.yahoo.com/

=
Instruções para entrar na lista, sair da lista e usar a lista em
http://www.mat.puc-rio.br/~nicolau/olimp/obm-l.html
O administrador desta lista é [EMAIL PROTECTED]
=



[obm-l] Re : [obm-l] Dúvidas sobre duas questões de análise real!!!

2003-01-23 Por tôpico Paulo Santa Rita
Ola Robson e demais
colegas desta lista ... OBM-L,

1) Caro Robson. Se voce nao sabe, logo  vai ficar sabendo ( quando comecar a 
estudar Analise ) que esta serie e um tipico representante do que se chama 
uma SERIE CONDICIONALMENTE CONVERGENTE.

Existe um Teorema ( de Riemann ) de Analise  que afirma que um reordenamente 
dos termos destas series pode faze-la divergir OU convergir ( convergir para 
UM NUMERO REAL QUALQUER, dado ). Claramente que este Teorema fala pouco ...

Seja A1, A2, ..., A3 um PA, isto e, Ai - Ai-1 = K, K # 0. Entao, pelo 
Teorema de Leibniz ( da Analise ), (1/A1) - (1/A2) + (1/A3) - ... converge. 
Converge pra onde ?  Isso vai depender da PA.  No seu caso :

1 - (1/2) + (1/3) - (1/4) + (1/5) - ... = Ln(2)

Outro caso bem conhecido e :

S = 1 - (1/3) + (1/5) - (1/7) + (1/9) - ... = pi/4

E uma relacao bem conhecida e que :

1 + (1/2^2) + (1/3^2) + (1/4^2) + ... = (1/3!)*((4S)^2).
A serie acima e o valor da funcao zeta em 2.

Note que  a sequencia 1, 1/(2^2),1/(3^2),1/(4^2), ... e tal que
1/Ai  -  2/Ai+1  +  1/Ai+2 = K, K constante e diferente de zero, para 
qualquer i. Toda serie que satisfaz a relacao acima e tal que :

A1 - 2*A2 + 3*A3 - 4*A4 + 5*A5 - ...

Converge condicionalmente, conforme voce pode mostrar facilmente usando o 
Teorema de Leibniz a que me referi acima.

Se admitirmos que as series alternadas cujos modulos dos inversos dos seus 
termos sao uma PA constituem um dado, entao o problema dos inversos das PA2 
fica bem posto. Mais claramente, seja A1, A2, ..., An uma sequencia tal que 
( K e S dados ) :

1) 1/Ai - 2/Ai+1 + 1/Ai+2 = K = constante nao nula, independente de i
2) A1 - 2*A2 + 3*A3 - 4*A4 + 5*A5 - 6*A6 + ... converge para S.

A serie abaixo converge para que numero real ? :

A1 + A2 + A3 + A4 + A5 + A6 + ...

Essa questao nao e simples. E uma forma diferente de abordar um problema 
resolvido apenas parcialmente pelo Euler.




2) Problema tipico de Introducao a Analise, que esta em todo livro desta 
categoria. Todo mundo mundo que estudou Analise fez este ou outro muito 
parecido. O enunciado esta um pouco confuso, mas sai assim : Suponha inf B  
sup A. Moste que isto conduz a um absurdo.
Logo sup A = inf B

Voce aqui estara usando o AXIOMA DO SUPREMO, tambem conhecido como AXIOMA DO 
COMPLETAMENTO : Todo conjunto de numeros reais limitado superiormente admite 
um supremo.

Deste axioma voce DEDUZ a existencia do infimo, vale dizer, postular o 
axioma do supremo implica no TEOREMA DO INFIMO. Mas nao existe nenhum 
razao forte para esta preferencia ...

Voce pode postular um AXIOMA DO INFIMO e deduzir o TEOREMA DO SUPREMO ( 
basta multiplicar por -1 e fazer um raciocinio bobo cheio de implicacoes 
obvias ). Um corpo ordenado no qual vale este axioma e um CORPO ORDENADO 
COMPLETO. Os numeros reais.

Ha autores que POSTULAM a existencia de um corpo ordenado completo, tais 
como o Prof Elon Lima ( Curso de Analise, Vol. 1). Todavia, historicamente, 
foi a percepcao do AXIOMA DO SUPREMO pelo Dedekin que permitiu CONSTRUIR um 
tal corpo, que alias pode ser construido por diversos outros caminhos ...

O fato de existir diversas construcoes de tais corpos ( ordenados e 
completos ) nao complica ... Dois  de tais corpos sao necessariamente 
isomorfos, vale dizer, INDISTINGUIVEIS no que se refere aos axiomas que os 
definem.

Um Abraco
Paulo Santa Rita
5,1750,230103


  - Original Message -
  From: Robson Monteiro
  To: [EMAIL PROTECTED]
  Sent: Tuesday, January 14, 2003 5:23 PM
  Subject: Dúvidas sobre duas questões de análise real!!!


  Oi Pessoal estou com duas dúvidas(sobre quetões que encontrei 
no livro do Elon Lages-Análise Real) e gostaria de saber se alguém pode me 
ajudar:

1º)  Efetue explicitamente uma reordenação dos termos da série 1 - (1/2) + 
(1/3) - (1/4) + (1/5) - ...

2º)  Sejam A,B conjuntos não vazios de números reais, tais que x Pertence 
a A e y pertence a B, com (x=y). Prove que supA=infB. Prove que 
supA=infB, se e somente se, para todo Epsilon0 dado, podem-se obter x 
pertencente a A e y pertencente a B tais que: y- x=epsilon


_
MSN Hotmail, o maior webmail do Brasil.  http://www.hotmail.com

=
Instruções para entrar na lista, sair da lista e usar a lista em
http://www.mat.puc-rio.br/~nicolau/olimp/obm-l.html
O administrador desta lista é [EMAIL PROTECTED]
=



[obm-l] Re:[obm-l] Re: [obm-l] Re:[obm-l] Re: [obm-l] dúvidas

2002-05-09 Por tôpico rafaelc.l

É, a numero 7 do IME de análise combinatória foi 
realmente difícil por ser tbm muito enganosa... por 
exemplo: eu achei que caminhos fossem auqeles em que não 
se pudesse repetir nem voltar trechos, mas quando vi a 
resolução, percibi que estava completamente errado e 
ainda por cima, não entendi a resolução.

 
__
Quer ter seu próprio endereço na Internet?
Garanta já o seu e ainda ganhe cinco e-mails personalizados.
DomíniosBOL - http://dominios.bol.com.br


=
Instruções para entrar na lista, sair da lista e usar a lista em
http://www.mat.puc-rio.br/~nicolau/olimp/obm-l.html
O administrador desta lista é [EMAIL PROTECTED]
=



[obm-l] Re:[obm-l] Re: [obm-l] dúvidas

2002-05-05 Por tôpico rafaelc.l


Interessante vc ter se baseado na letra A para fazer a 
letra B..não é a toa que elas estão na mesma questão. 
 Não sei se vc concorda comigo, mas essa questão foi pra 
mim a mais difícil da prova e esse tipo de questão que 
chamam de álgebra especulativa eu acho realmente mais 
complicado que o resto em matemática. Vc concorda?

 
__
Quer ter seu próprio endereço na Internet?
Garanta já o seu e ainda ganhe cinco e-mails personalizados.
DomíniosBOL - http://dominios.bol.com.br


=
Instruções para entrar na lista, sair da lista e usar a lista em
http://www.mat.puc-rio.br/~nicolau/olimp/obm-l.html
O administrador desta lista é [EMAIL PROTECTED]
=



[obm-l] Re:[obm-l] Re: [obm-l] dúvidas

2002-05-04 Por tôpico rafaelc.l

valeu Daniel. Mas de onde vc tira essas fatorações? isso 
eu nunca aprendi, aliás, esse tipo de questão abrange 
álgebra básica, que não tem em nenhum livro..não é?

 
__
Quer ter seu próprio endereço na Internet?
Garanta já o seu e ainda ganhe cinco e-mails personalizados.
DomíniosBOL - http://dominios.bol.com.br


=
Instruções para entrar na lista, sair da lista e usar a lista em
http://www.mat.puc-rio.br/~nicolau/olimp/obm-l.html
O administrador desta lista é [EMAIL PROTECTED]
=



[obm-l] Re: [obm-l] dúvidas

2002-05-04 Por tôpico Adriano Almeida Faustino

(IME-96)
sendo T(k+1) o termo de ordem (k+1)
T(k+1) = C (65,k) * (1/3)^k * 1^(65-k) = 1/3^k * C(65,k)

se T (k+1) é o maior :  T (k+1) = T(k)
1/3^k * C(65,k) = 1/3(k-1) * C(65,(k-1))
efetuando as contas : 1/3k = 1/(66-k)
k = 66/4 = 16,5

e também : T(k+1) = T(k+2)
1/3^k * C(65,k) = 1/3^(k+1) * C(65,(k+1))
1/(65-k) = 1/(3(k+1))
k = 62/4 =15,5
15,5 = k = 16,5   - k=16 , já que k é inteiro

T(16+1) =T(17) = C(65,16)/3^16

Adriano.
From: rafaelc.l [EMAIL PROTECTED]
Reply-To: [EMAIL PROTECTED]
To: [EMAIL PROTECTED]
Subject: [obm-l] dúvidas
Date: Fri,  3 May 2002 15:31:10 -0300


(IME-96)Determine o termo máximo do desenvolvimento da
expressão:
 (1+1/3)^65



(IME-2001) a) Sejam x,y,z números reais positivos, prove
que:
 x+y+z/3 maior ou igual a (x.y.z)^1/3

Em que condições a igualdade se verifica?

  b) Seja um paralelepípedo de lados a,b e c e área total
S.Ache o volume máximo desse paralelepípedo em função de
S. Qual a relação entre a, b e c para que esse volume
seja máximo? demonstre seus resultados.


OBS: ponham o maior número de soluções possíveis.


Obrigado


__
Quer ter seu próprio endereço na Internet?
Garanta já o seu e ainda ganhe cinco e-mails personalizados.
DomíniosBOL - http://dominios.bol.com.br


=
Instruções para entrar na lista, sair da lista e usar a lista em
http://www.mat.puc-rio.br/~nicolau/olimp/obm-l.html
O administrador desta lista é [EMAIL PROTECTED]
=


_
Una-se ao maior serviço de email do mundo: o MSN Hotmail. 
http://www.hotmail.com

=
Instruções para entrar na lista, sair da lista e usar a lista em
http://www.mat.puc-rio.br/~nicolau/olimp/obm-l.html
O administrador desta lista é [EMAIL PROTECTED]
=



[obm-l] Re: [obm-l] dúvidas

2002-05-04 Por tôpico Adriano Almeida Faustino


(IME-2001)
a)sendo a e b reais positivos temos que :
(a^(1/2)-b(^1/2))^2 = 0
temos que a+b = 2(ab)^(1/2)
fazendo a = (c+d)/2   , b= (e+f)/2

(c+d+e+f)/4 = ( (c+d)/2 *(e+f)/2 )^(1/2)

utilizando a desigualdade entre as médias provada no começo:
(c+d+f+e)/4 = (cdef)^(1/4)

fazendo um termo qualquer como a média arit. dos outros  temos  que:
( (c+d+e)+ (c+d+e)/3 )/4 = ( cde(c+d+e)/3 )~(1/4)

(c+d+e)/3 = ( cde(c+d+e)/3 )^(1/4)
elevando a quarta pot. e simplificando nos dois lados temos:

(c+d+e)/3 = (cde)^(1/3)

b)comoS = 2(ab+bc+ac), fazendo x=ab ,y=ac, z=bc  temos:
(ab+ac+bc)/3 = ( (ab)(ac)(bc) )^(1/3)

S/6 = ( (abc)^2) ^(1/3)
S/6 = ( V^2 ) ^(1/3)
V = S/6 ( (S/6)^(1/2) )
então o valor máximo de V é igual a S/6 ( (S/6) ^(1/2) )
o que implica que a média arit. é igual a média geom.
então a=b=c
ou seja o paral. é um cubo.
Adriano
From: rafaelc.l [EMAIL PROTECTED]
Reply-To: [EMAIL PROTECTED]
To: [EMAIL PROTECTED]
Subject: [obm-l] dúvidas
Date: Fri,  3 May 2002 15:31:10 -0300


(IME-96)Determine o termo máximo do desenvolvimento da
expressão:
 (1+1/3)^65



(IME-2001) a) Sejam x,y,z números reais positivos, prove
que:
 x+y+z/3 maior ou igual a (x.y.z)^1/3

Em que condições a igualdade se verifica?

  b) Seja um paralelepípedo de lados a,b e c e área total
S.Ache o volume máximo desse paralelepípedo em função de
S. Qual a relação entre a, b e c para que esse volume
seja máximo? demonstre seus resultados.


OBS: ponham o maior número de soluções possíveis.


Obrigado


__
Quer ter seu próprio endereço na Internet?
Garanta já o seu e ainda ganhe cinco e-mails personalizados.
DomíniosBOL - http://dominios.bol.com.br


=
Instruções para entrar na lista, sair da lista e usar a lista em
http://www.mat.puc-rio.br/~nicolau/olimp/obm-l.html
O administrador desta lista é [EMAIL PROTECTED]
=


_
Envie e receba emails com o Hotmail no seu dispositivo móvel: 
http://mobile.msn.com

=
Instruções para entrar na lista, sair da lista e usar a lista em
http://www.mat.puc-rio.br/~nicolau/olimp/obm-l.html
O administrador desta lista é [EMAIL PROTECTED]
=



[obm-l] Re: [obm-l] dúvidas em análise combinatória

2002-04-30 Por tôpico rafaelc.l


Mas no caso do problema 1, pq tem de terminar em 12, 16, 
24, 36 ou 56?






 2) Vou trocar os fatores para p, q, r, s
  Os divisores sao da forma (p^a) * (q^b) * (r^c) * 
(s^d)  , cada um dos 
 números a, b, c, d podendo ser 0 ou 1. Há 2 modos de 
escolher o valor de 
 a,  2 modos de escolher o valor de b,...
 A resposta é 2x2x2x2=16.
 1) O final do número só pode ser 12, 16, 24, 36, 56. Há 
5 modos de 
 escolher os dois últimos algarismos. Depois disso, 
haverá 4 modos de 
 escolher o primeiro algarismo e 3 modos de escolher o 
segundo.
 A resposta é  5x4x3=60.
 
 rafaelc.l wrote:
 
 1)qual é o total de números múltiplos de 4, com quatro 
 algarismos distintos, que podem ser formados com os 
 algarismos 1,2,3,4,5 e 6?
 
 2)Seja o número natural N=p1.p2.p3.p4 onde p1,p2,p3 e 
p4 
 são fatores naturais primos distintos. Qual o número 
de 
 divisores naturais de N?
 
 
 Me ajudem a resolver os 2 problemas acima..
  
   Obrigado
 
  
 
_
_
 Quer ter seu próprio endereço na Internet?
 Garanta já o seu e ainda ganhe cinco e-mails 
personalizados.
 DomíniosBOL - http://dominios.bol.com.br
 
 
 
=

 Instruções para entrar na lista, sair da lista e usar 
a lista em
 http://www.mat.puc-rio.br/~nicolau/olimp/obm-l.html
 O administrador desta lista é [EMAIL PROTECTED]
 
=

 
 
 
 
 
==
===
 Instruções para entrar na lista, sair da lista e usar a 
lista em
 http://www.mat.puc-rio.br/~nicolau/olimp/obm-l.html
 O administrador desta lista é [EMAIL PROTECTED]
 
==
===
 

 
__
Quer ter seu próprio endereço na Internet?
Garanta já o seu e ainda ganhe cinco e-mails personalizados.
DomíniosBOL - http://dominios.bol.com.br


=
Instruções para entrar na lista, sair da lista e usar a lista em
http://www.mat.puc-rio.br/~nicolau/olimp/obm-l.html
O administrador desta lista é [EMAIL PROTECTED]
=



Re: [obm-l] Re: [obm-l] dúvidas em análise combinatória

2002-04-30 Por tôpico Augusto César Morgado



*Um numero eh divisivel por 4 se e somente se o numero formado pelos dois
ultimos algarismos for divisivel por 4. Tem que terminar em 12,16,24,32,36,52,
56 ou 64. Ha 8 modos. A resposta eh 8x4x3=96
*Se n=(...cab), n= 100x(...c)+(ab)

rafaelc.l wrote:
GVD137$[EMAIL PROTECTED]">
  Mas no caso do problema 1, pq tem de terminar em 12, 16, 24, 36 ou 56?
  
2) Vou trocar os fatores para p, q, r, s Os divisores sao da forma (p^a) * (q^b) * (r^c) * 

(s^d)  , cada um dos 

  nmeros a, b, c, d podendo ser 0 ou 1. H 2 modos de 
  
  escolher o valor de 
  
a,  2 modos de escolher o valor de b,...A resposta  2x2x2x2=16.1) O final do nmero s pode ser 12, 16, 24, 36, 56. H 

5 modos de 

  escolher os dois ltimos algarismos. Depois disso, 
  
  haver 4 modos de 
  
escolher o primeiro algarismo e 3 modos de escolher o 

segundo.

  A resposta   5x4x3=60.rafaelc.l wrote:
  
1)qual  o total de nmeros mltiplos de 4, com quatro algarismos distintos, que podem ser formados com os algarismos 1,2,3,4,5 e 6?2)Seja o nmero natural N=p1.p2.p3.p4 onde p1,p2,p3 e 


p4 

  
so fatores naturais primos distintos. Qual o nmero 


de 

  
divisores naturais de N?Me ajudem a resolver os 2 problemas acima.. Obrigado

_

_

  
Quer ter seu prprio endereo na Internet?Garanta j o seu e ainda ganhe cinco e-mails 


personalizados.

  
DomniosBOL - http://dominios.bol.com.br

=



  
Instrues para entrar na lista, sair da lista e usar 


a lista em

  
http://www.mat.puc-rio.br/~nicolau/olimp/obm-l.htmlO administrador desta lista  [EMAIL PROTECTED]

=



  




=

  Instrues para entrar na lista, sair da lista e usar a 
  
  lista em
  
http://www.mat.puc-rio.br/~nicolau/olimp/obm-l.htmlO administrador desta lista  [EMAIL PROTECTED]

=
 __Quer ter seu prprio endereo na Internet?Garanta j o seu e ainda ganhe cinco e-mails personalizados.DomniosBOL - http://dominios.bol.com.br=Instrues para entrar na lista, sair da lista e usar a lista emhttp://www.mat.puc-rio.br/~nicolau/olimp/obm-l.htmlO administrador desta lista  [EMAIL PROTECTED]=